Sei sulla pagina 1di 50

C

2
H
A
P Foreign Exchange Risk Management
T
E
R

Coverage of FOREX in Past Examinations:


Year May-2012 Nov-2011 May- Nov- May-2010 Nov- June- Nov- May-
2011 2010 2009 2009 2008 2008
SFM 8+10(Theory) 11+8(With
(New) --- 8 4 12 12 20 NA
=18 derivative)=19
MAFA 8 (With
(Old)
NA NA NA 4
derivative)
7 6 12 8

Contents:
1. Introduction
2. Major foreign currency
3. Regulator of forex market in India
4. Types of Forex Market.
5. How transaction takes place in forex?
6. Bid, Ask and Spread
7. Exchange rate quotation (One way quote & Two way quote)
8. Interpretation of exchange rate
-Quoted in form “/€”
-Quoted in compressed form
-Few other cases
9. Types of transaction in Forex
10. Forward premium & Discount:
11. Direct quote & Indirect quote
12. Loss/gain due to Forward cover; Premium/Discount on currency
13. Swap point/Forward margin/Forward Spot differential.
14. Exchange margin
15. General concept question
16. Benefit of Early Payment
17. Cross Rates of Foreign Exchange
18. Hedging technique:
-Hedging through Forward market
-Hedging through Money (cash) Market Operation:
-Leading/Lagging
-Rupee Roll-Over-forward Contracts
-Currency option (Refer derivative)
-Currency future (Refer derivative)
19. Exchange rate determination Theory
-Interest Rate Parity (IRPT)
-Purchasing Power Parity OR Inflation rates parity
20. Nostro A/C ; Vostro A/C & LORO Account
-Maintaining Exchange position and Nostro A/C
-Transaction through demand draft (DD)

www.fmguru.org CA Nagendra P a g e | 2.1


Strategic Financial Management

-Transaction through Bills Receivable (B/R):


21. Cancellation of forward Contract:
22. Modification in forward contract:
23. Early Delivery of Currency
24. Arbitrage
-Geographical Arbitrage
-Cover interest Arbitrage
25. LC (Letter of Credit) Arrangement
26. Parallel Loan
27. Cash management between Holding and Subsidiary
28. Netting of foreign exchange
29. FOREX with Derivative
Introduction
In this chapter we will study how to minimize the loss /risk which will arise due to the adverse change in exchange
rate of foreign currency. Normally, Payment will be receivable or payable at some future date and exchange rate will
not be same at that day. As a result loss/gain will arise.
The exchange rate is the relationship between two currencies at a given point of time.
For example: $ 1 =  46
One way quote
£ 1 =  77
$1 =  46 -  46.5
Two way quote
£ 1 =  70 -  70.5

Major Foreign Currency:


Country/territory (Sub Country) Currency ISO Code* Sign/symbol Fractional Unit
1. Kuwait Kuwaiti Dinar KWD --- 1000 fils
2. United Kingdom (UK) British pound GBP £ 100 penny
3. Eurozone Euro EUR € 100 Cent
[Total 28 country using Euro currency]
4. USA US dollar USD $ 100 Cent
5. Switzerland Swiss Franc CHF Fr 100 Rappen
6. India Indian Rupee INR  100 Paisa
7. Nepal Nepali Rupee NPR Rs 100 Paisa
8. Japan Japanese Yen JPY ¥ 100 Sen
* ISO Code: Internationally Standardized three letter abbreviation
*Note: Earlier Germany uses Deutsche mark (DEM or DM) (Also known as “Mark” or “German Mark”) Currency. But Now Germany
using “Euro” (€) currency
Earlier France was using French Franc (FRF) currency. But now France are using “Euro” (€) currency
In question we will still found DEM and FRM currency.
Regulator of Forex Market in India
All foreign exchange transaction made in India is being regulated either by RBI or FEDAI. In other word, we can say
that a person doing transaction in foreign exchange has to follow RBI rule and FEDAI rule.

Regulator of Forex Market

RBI FEDAI
(Reserve Bank Of India) (Foreign Exchange Dealers Association)

Established in accordance with the Established under Section-25 of


provisions of RBI Act 1934 Companies Act 1956
Manage FEMA Act 1999 & maintain Regulate Inter-bank foreign exchange
Foreign exchange market business
Website: www.rbi.org.in  Website: www.fedai.org.in

P a g e | 2.2 CA. Nagendra www.fmguru.org


FOREX

Types of Forex Market:

Forex Market

Inter bank/Wholesale Mkt. Retail Market

Under inter Bank Market one Bank Under retail Market Bank deal with Customer.
deal with other bank Exchange rate of retail market is known as
Exchange rate of Inter-bank Merchant rate.
market is known as Inter Bank rate. A customer can buy or sell currency only at
Only bank can deal at Inter Bank merchant rate.
rate. Merchant rate is derived from interbank rate
by adding or deducting exchange margin.

Forex Market is a world’s largest market whose average daily turnover is around $ 4 trillion.
1 million = 10 Lacs
1 billion = 1000 million
1 trillion = 1000 billion

How transaction takes place in FOREX


Each party want to trade and deal in own currency but the invoice can be made in single currency. Hence the agreed
currency would be the home currency, foreign currency or any other currency.
Remember that, a person can’t keep foreign currency with himself. If anyone receives foreign currency he has to sell it
to forex dealer/bank and similarly if he has to pay foreign currency, he will buy foreign currency form forex
dealer/Bank.

Option-I when invoice made in home currency

Export to UK co.
USA Company($) 1 UK Company (£)
Inflow at 3 Month Invoice Amount: Outflow at 3 Month
Option A: $ 10,000
(Credit period 3M)

3 2B 2A

$ £
UK co. make payment at 3
month time in ‘$’ Currency UK Bank
Buy £, sell $

Risk of currency fluctuation lies on UK company


as UK Co has to convert £ into $ at 3 month time.

When invoice is in “$”: chain in steps, made in figure 12A2B3. (at end, USA co. get money in $)
Analysis for USA co: when invoice is in home currency ($)  No risk on USA co.  No need to hedge foreign currency
receipt and hence no need to study FOREX management.
Analysis for UK co: when invoice is in foreign currency ($)  Risk lies on UK co.  UK Company need to hedge
foreign currency payment.

www.fmguru.org CA. Nagendra P a g e | 2.3


Strategic Financial Management

Option-II [When Invoice made in foreign currency]

Export to UK co.
USA Company($) 1 UK Company (£)
Inflow at 3 Month Invoice Amount: Outflow at 3 Month
Option A: £ 5,000
(Credit period 3M)

3B 3A 2

$ £
UK co. make payment at 3
USA Bank month time in ‘£’ Currency
Buy £, sell $

Risk of currency fluctuation lies on USA company


as USA Co has to convert £ into $ at 3 month time.

When invoice is in “£”: chain in steps, made in figure 123A3B. (at end, USA co. get money in $)

Analysis for USA co: when invoice is in foreign currency (£)  Risk lies on USA co.  Need to hedge foreign currency
receipt.

Analysis for UK co: when invoice is in home currency (£)  No risk on UK co.  No need to hedge home currency
payment hence no need to study FOREX management.

Option-III [When invoice made in other country currency]

Export to UK co.
USA Company($) 1 UK Company (£)
Inflow at 3 Month Invoice Amount: Outflow at 3 Month
Option A: € 5,000
(Credit period 3M)

4B 4A 3 2B 2A

$ € € £
UK co. make payment at 3
USA Bank month time in ‘€’ Currency USA Bank
Buy €, sell $ Buy £, sell €

Risk of currency fluctuation lies on both USA company & UK company as


both Co has to convert one currency into another currency at 3 month time.

When invoice is in “€”: chain in steps, made in figure 12A2B34A4B. (at end, USA co. get money in $)

Analysis for USA co: when invoice is in foreign currency (€)  Risk lies on USA co.  Need to hedge foreign currency
receipt.

Analysis for UK co: when invoice is in foreign currency (£)  Risk lies on UK co. also  Need to hedge foreign
currency payment.

P a g e | 2.4 CA. Nagendra www.fmguru.org


FOREX

Conclusion:
From the above analysis it is clear that Hedging is done only when the invoice is being made in foreign currency.
Hedging is not required when invoice is made in home currency.
If question ask for any measures of hedging, it mean the currency payable or receivable is always foreign currency.
Hence do not get confused to recognize which currency is foreign currency and which currency is home currency.
Just identify the currency in which invoice is being made (i.e. currency in which amount payable or
receivable), and conclude that invoice currency is always foreign currency. On basis of this currency identify
the company who has to take corrective action to minimize loss.

Bid, Ask & Spread


Bid rate (Bank’s buying rate): Bid rate is the rate at which bank buys left hand currency.
Ask rate (Bank’s selling rate): Ask rate is the rate at which bank sells left hand currency.
Note: Ask rate will always be greater than bid rate (i.e. Ask rate > Bid rate)
Spread: The difference between Ask and Bid rates is called the spread.
Mathematically, Spread = Ask rate – Bid rate

% Spread = 100

Exchange rate quotation


(i) One way quote:
When bid rate (i.e. Buy rate) and Ask rate (i.e. sale rate) are same, it is the one way quote. Practically one
way quote is not possible.
For example: £ 1 = $ 1.50
It means bank will buy left hand currency (i.e. £) at $ 1.50 and also sale left hand currency (i.e. £) at $ 1.50.
(ii) Two way quote:
When bid rate (i.e. Buy rate) and Ask rate (i.e. sale rate) are different, it is the two way quote.
For example: £ 1 = $ 1.50 - $ 1.55

Bid rate Ask rate

Buying rate selling rate


Of bank of bank
Bid rate is always less than Ask rate
It means bank will buy left hand currency (i.e. £) at $ 1.50.
and bank will sale left hand currency (i.e. £) at $ 1.55

How to apply two way quote rate, for converting one currency into other currency?
1. Identify amount payable or receivable?
For example: assume invoice amount is $ 1,00,000/- which will be payable in 3 month time.

2. Choose applicable Bid or Ask rate


Take the given exchange rate and draw the following diagram and identify what will do bank for left hand
currency (think left hand currency is apple. Here apple is £ so circle £ in figure). Bank will buy £. Bid rate is applicable.

Rate: £ 1 = $ 1.50 – 1.55


Buy £ Sale $
Applicable Rate Bank

Here amount payable is $ 1,00,000/-, means customer have to buy $ from bank. We made diagram on the
basis of our requirement but think what will do bank for ‘£’ currency and not for ‘$’ currency because left
hand currency given in rate is ‘£’ currency.
Conclusion: Apply Bid rate i.e. $ 1.5 for conversion of $1,00,000.

www.fmguru.org CA. Nagendra P a g e | 2.5


Strategic Financial Management

3. Convert one currency into other using selected rate.


Use unitary method to convert one currency into other.
Here, £ 1 = $ 1.50 [pick only bid rate which is selected in step-2]
$1.50 = £ 1
$1,00,000 = £
.5
= £ 66,666.66

Interpretation of exchange rate quoted in form “/€”

“/€ means: Normally interpret


this one. Because
95% question’s
Exchange rate is
“€ per” “ Per €” quoted in this form.
In Practical world In Academic word

Example: 1. In practical life we will find quotation, in this


Exchange Rate: /€ 60.00 / 60.25. What does it mean? form up to 4 decimal places. “0.0001” value is
Normal interpretation: It means € =  60.00 – 60.25 or, known as 1 Pip.
 It means: 1 = € 60.00 – 60.25
Which one we prefer?
We have to apply our common sense on the basis of strength of the currency to decide this. Stronger currency
have lower figure
Here, Normal interpretation is correct. Because € is stronger currency than. Hence, € 1 =  60.00 – 60.25

Example-2:
Given exchange rate is /¥ = 2.25/2.40 What does it mean?

Normal interpretation: 1¥ = 2.25/2.40


Hence it means 1  = ¥ 2.25/2.40

Strength wise currency list for some important currency: (strongest to weakest currency sequence):
1. Kuwaiti Dinar-KWD 195
2. Pound –GBP (£) 85
3. Euro-EUR (€) 70
4. Swiss Franc (CHF) 58
5. US Dollar-USD ($) 55
6. Indian Rupee-INR () 1
7. Japanese Yen-JPY (¥) 0.70

Example 3:
Given Rate: ( per €): 0.0165 – 0.0175 . What does it mean?

In this case we do not need any interpretation because question specifically written  per €.
It mean: € =  0.0165 – 0.0175
Solve question assuming per € even rate is illogical, because rate is specifically written as  per € and we know that
specific provision overrides general.

Alternatively one can use this rate as per logic i.e. € per  on the basis of strength. Both will be correct – i.e. on the
basis of strength and another, on the basis of specific information.

P a g e | 2.6 CA. Nagendra www.fmguru.org


FOREX

Interpretation of exchange rate Quoted in compressed form:


Exapmle-1:
Given exchange rate is: 1$ = 45.4245/50
It means rate is: 1$ = 45.4245/ 45.4250

Example-2:
Given exchange rate is 1$ = 45.5189/12
It means rate is: 1$ = 45.5189/ 45.5212

Interpretation of exchange rate in few other cases:


Suppose given exchange rate quotation is:
Spot rate (€ per £) : .782
1 month forward rate (€ per £) : .7829

It means spot rate is: £ = € ( .782 -0.0002) / (1.7820+0.0002)


= € .78 8 / .7822
(Note: It doesn’t mean that bid rate is 1.7820 and Ask rate is (1.7820+0.0002)
month forward rate is: £ = € ( .7829-0.0003) / (1.7829+0.0003)
= € .7826 / .7832

Type of Transaction in Forex


Normally, we use spot rate transaction for current settlement. And Forward transaction for future settlement.

Type of Transaction

Ready transaction Value tom transaction Spot transaction Forward transaction

Transaction entered Transaction entered Transaction entered Transaction entered


today for same day today for next working today for (T + 2) today for Future day
settlement. day settlement. working day settlement.
settlement.

Diagrammatic representation:

Indian Import of goods


US supplier
importer

 $ Payment in $

Settlement Date:
Bank Ready Transaction: Immediate
Value tom: Next Business Day
Spot Transaction: T+2 Bus. Day
Agreement Date: Today for all types of transaction Forward Transaction: Future day
Applicable rate: Ready rate, Value tom rate, Spot rate, forward
rate as the case may be.

www.fmguru.org CA. Nagendra P a g e | 2.7


Strategic Financial Management

Note:
In all transaction Exchange rate is decided today and settlement being made at different date. For different type of
transaction different rate is applicable. The exchange rate for each transaction is not same.
Normally, we use spot rate transaction for current settlement. And Forward transaction for future Settlement.

Forward premium & Discount:


If the currency is costlier in future as compare to spot it is said to be at a premium.
If the currency is cheaper in future as compare to spot it is said to be at a discount.
Concept example for calculation of premium (increase) and discount(decrease):
Ex-(1)
100 High Price

%increase = 100 % Decrease = 100


= 25% = 20%

80 Low Price

To calculate % increase, we take base of 80.


To calculate %decrease, we take base of 100.
Hence, % increase and % decrease will not be same because of different base used.

Ex-(2)
Today Future date
$ will be on Premium
$ 1 = 50 Case-I Premium amount = 5.55
1 $ = 55.55
% premium = 100 = 11.11%

Case-II
$ will be on discount
1 $ = 47.62 Discount amount = 2.38
% Discount = 100 = 4.76%

Ex-(3) All rates are same as Ex-2, but quoted in indirect form.

Today Future date


 will be on discount
Case-I Discount = 0.002 Apple
1 = 0.02 $
1 = 0.018 $ % Discount = 100 = 10%

Case-II  will be on premium


Premium amount = 0.001 Apple
1 = 0.021 $
% premium = 100 = 5%

In case - I  $ will be on premium and will be on discount. But Rate is different (i.e.11.11% & 10%)
Inc case -II$ will be on Discount and will be on Premium  But Rate is different (i.e. 4.76% & 5%

P a g e | 2.8 CA. Nagendra www.fmguru.org


FOREX

How we will calculate premium/Discount on $ using Ex-3’s Indirect quote:


Case-I: premium on dollar = 100 % premium = 11.11%
Case-II: Discount on Dollar = 100  % Premium = 4.76%

How to calculate forward premium or discount when spot rate and forward rate is given:
 First decide on which currency you have to calculate premium rate of discount rate.
 Check the quotation, whether it is direct quote or indirect quote for the decided currency.

For Direct quote For Indirect quote


Annual Forward premium/discount = Annual Forward premium/discount =

If calculated value is +ve Means premium If calculated value is +ve Means premium
If calculated value is –ve  Means discount If calculated value is –ve  Means discount

How to calculate forward rate when forward premium & Discount given?
 Take spot rate. (say $1 =  45)
 Multiply by (1+premium rate) to the other currency
(i.e. if premium is on $ then multiply in  amount or vice versa). Or
 Multiply by (1- discount rate) to other currency
(i.e. if discount is on $ then multiply in amount or vice versa).

Direct quote & Indirect quote


Direct quote:
A direct quote is the home currency price for one unit foreign currency.
Example: $1 =  48 is a direct quote for an Indian
Indirect quote:
An indirect quote is the foreign currency price for one unit of home currency.
Example:  1 = $ 0.0208 is an indirect quote for an Indian.

Note: To solve practical problem, we will think direct quote/indirect quote for any one currency and not for any
country.
$1 =  48 Direct quote for $ and Indirect quote for 

Converting direct quote into indirect quote:


Conversion for one way quote
Direct quote = OR Indirect quote =

Example: Direct quote for $: $ 1 =  45


Indirect quote for $: 1=$

www.fmguru.org CA. Nagendra P a g e | 2.9


Strategic Financial Management

Conversion for two way quote:


Example: $1 =  45 / 45.50 1= / i.e.  1 = 0.0222/0.0219
Here, Because of Bid rate > Ask rate, above conversion is not correct.
Hence Use following:
Direct quote: $1 =  45 / 45.50

Indirect quote:  1 = / i.e. 1 = 0.0219 / 0.0222 (Bid rate < Ask rate)

Hedging through forward Market:


For hedging (safeguard) future receipt or future payment one has to enter forward contract (i.e. today’s contract for future delivery) at
forward rate.
At due date convert one currency into other currency at the agreed rate the rate at which forward contract is made, doesn’t matter
what is the actual rate on due date.

Calculation of gain/Loss
Suppose a person has to buy 1 Kg Apple.

Today 6 month

Forward Contract for buying 1 Apple =  45


1 Apple = 40
Expected rate 1Apple = 46

 Expected loss = 6 Actual rate 1 Apple = 47


 Actual loss if forward cover not taken = 7
 Loss due to forward cover = 5 (In this case Actual rate/expected rate at 6 month time should not be given.)

 How expected loss can be hedged by forward cover?


By entering forward cover expected loss can be reduce from 6 to 5.

 Loss due to forward cover if 6 month rate is 47 = 2 Gain If Rate at 6 month time is given then
 Loss due to forward cover if 6 month rate is 46 = 1 Gain calculate loss/gain using forward rate
and 6 month time rate.

Loss/gain due to Forward cover; Premium/Discount on currency


Question No. - 1A (Nov.-2004-Old-4 Marks)
Excel Exporters are holding an Export bill in United States Dollar (USD) 1,00,000 due 60 days hence. They are
worried about the falling USD value which is currently at  45.60 per USD. The concerned Export Consignment
has been priced on an Exchange rate of  45.5 per USD. The Firm’s Bankers have quoted a 6 -day forward rate
of  45.20.
Calculate:
(i) Rate of discount quoted by the Bank
(ii) The probable loss of operating profit if the forward sale is agreed to.
Ans: (i) Annual Discount = 5.38%; (ii) Probable loss =  30,000

P a g e | 2.10 CA. Nagendra www.fmguru.org


FOREX

Question No. -1B [Nov-2003-Old-10 Marks]


ABC Co. has taken a 6 month loan from their foreign collaborators for US Dollars 2 millions. Interest payable on
maturity is at LIBOR plus 1.0%. Current 6-month LIBOR (London Interbank Offering rate) is 2%.
Enquiries regarding exchange rates with their bank elicit the following information:
Spot USD 1  48.5275
6 months forward  48.4575
(i) What would be their total commitment in Rupees, if they enter into a forward contract?
(ii) Will you advise them to do so? Explain giving reasons.
Ans: (i) Value of Commitment =  9,83,68,725
(ii)If own expectation- that dollar will depreciate more than quoted- do nothing; If firm has no specific
view – cover the exposure; it would be unwise to expect continuous depreciation of the dollar.
The US Dollar is a stronger currency than the Indian Rupee based on past trends Hence it would
be advisable to enter forward contract.

Question No. - 1C [Study Mat.]


Fleur du lac, a French co, has shipped goods to an American importer under a letter of credit arrangement, which
calls for payment at the end of 90 days. The invoice is for $1, 24,000. Presently the exc hange rate is 5.70 French
francs to the $ if the French franc were to strengthen by 5% by the end of 90 days, what would be the
transactions gain or loss to exporter in French francs?
If it were to weaken by 5%, what would happen?
Ans: If Strengthen by 5%: Loss = 33,604 FRF
If Weaken by 5%: Gain = 37,200 FRF

Question No. - 1D [Nov-2003-old-6 Marks]


A company operating in Japan has today effected sales to an Indian company, the payment being due 3 months from
the date of invoice. The invoice amount is 108 lakhs yen. At today's spot rate, it is equivalent to  30 lakhs. It is
anticipated that the exchange rate will decline by 10% over the 3 months period and in order to protect the yen
payments, the importer proposes to take appropriate action in the foreign exchange market. The 3 months forward
rate is presently quoted as 3.3 yen per rupee. You are required to calculate the expected loss and to show how it can
be hedged by a forward contract.
Ans: Expected exchange loss = 3.33 Lakh; If hedged by a forward contract expected
loss = 2.73 Lakh; i.e. reduced by 0.6 lakh

Question No. - 1E [Study Mat.]


A UK company, is due to receive 5,00,000 Northland dollars in six month’s time for goods supplied. The company
secedes to hedge its currency exposure by using the forward market. The spot rate of exchange is 2.5 Northland
dollars to the pound. The forward rate of exchange is 2.5354 Northland dol lars to the pound.
Calculate how much UK company actually gains or losses as a result of the hedging transaction if, at the end of
the six months, the pound, in relation to the Northland dollar, has (i) gained 4%, (ii) lost 2% or (iii) remained
stable.
Ans: (i) Gain = £ 4,900; (ii) Loss = £ 6,874; (iii) Loss = £ 2,792.
Question No. - 1F [May-2008-old-8 Marks]
A company is considering hedging its foreign exchange risk. It has made a purchase on 1st. January, 2008 for which it
has to make a payment of US $ 50,000 on September 30, 2008. The present exchange rate is 1 US $ =  40. It can
purchase forward 1 US $ at  39.
The company will have to make a upfront premium of 2% of the forward amount purchased. The cost of funds to the
company is 10% per annum and the rate of Corporate tax is 50%. Ignore taxation. Consider the following situations
and compute the Profit/Loss the company will make, if it hedges its foreign exchange risk:
(i) If the exchange rate on September 30, 2008 is  42 per US $.
(ii) If the exchange rate on September 30, 2008 is  38 per US $.
Ans: (i) Due to forward cover, Gain = 1,08,075; (ii) Due to forward cover, Loss = 91,925

www.fmguru.org CA. Nagendra P a g e | 2.11


Strategic Financial Management

Question No. - 1G [Nov-2007-old-8 Marks]


Following information relates to AKC Ltd. which manufactures some parts of an electronics device which are exported
to USA, Japan and Europe on 90 days credit terms.
Cost and Sales information: Japan USA Europe
Variable cost per unit 225 395 510
Export sale price per unit Yen 650 US$10.23 Euro 11.99
Receipts from sale due in 90days Yen78,00,000 US$1,02,300 Euro 95,920
Foreign exchange rate information:
Yen/ US$/ Euro/
Spot market 2.417-2.437 0.0214-0.0217 0.0177-0.0180
3 months forward 2.397-2.427 0.0213-0.0216 0.0176-0.0178
3 months spot 2.423-2.459 0.02144-0.02156 0.0177-0.0179
Advice AKC Ltd. by calculating average contribution to sales ratio whether it should hedge it’s foreign currency risk or
not.
Ans: (i) If foreign exchange risk is hedged, ratio = 19.56%
(ii) If foreign exchange risk is not hedged, ratio = 19.17%

Question No. -1H [RPT-May-2011]


Arnie operating a garment store in US has imported garments from Indian exporter of invoice amount of  138,00,000
(equivalent to $ 3,00,000). The amount is payable in 3 months. It is expected that the exchange rate will decline by 5%
over 3 months period. Arnie is interested to take appropriate action in foreign exchange market. The three month
forward rate is quoted at 44.50
You are required to calculate expected loss which Arnie would suffer due to this decline if risk is not hedged. If there is
loss, then how he can hedge this risk.
Ans: Expected loss = $15,789, By taking forward cover loss can reduce to $10,112

Question No. -1i [RTP-Nov-2010-Old]


An Automobile company in Gujarat exports its goods to Singapore at a price of SG$ 500 per unit. The company also
imports components from Italy and the cost of components for each unit is € 2 . The company’s CEO executed an
agreement for the supply of 20,000 units on January 01, 2010 and on the same date paid for the imported
components. The company’s variable cost of producing per unit is  1,250 and the allocable fixed costs of the
company are 1,00,00,000.
The exchange rates on 1 January 2010 were as follows
Spot /SG$ 33.00/33.04
/€ 56.49/56.56
Mr. A, the treasury manager of company is observing the movements of exchange rates on a day to day basis and has
expected that the rupee would appreciate against SG$ and would depreciate against €.
As per his estimates the following are expected rates for 30th June 2010
Spot /SG$ 32.15/32.21
/€ 57.27/57.32
You are required to find out:
(a) The change in profitability due to transaction exposure for the contract entered into.
(b) How many units should the company increases its sales in order to maintain the current profit level for the
proposed contract in the end of June 2010.
Ans: (a) Profit will decrease by 115,40,000 (ii) Company would increase its sale from 20,000 to 23,434 units

P a g e | 2.12 CA. Nagendra www.fmguru.org


FOREX

Question No. -1J [Nov-2009-New-12 Marks]


M/s Omega Electronics Ltd. exports air conditioners to Germany by importing all the components from Singapore.
The company is exporting 2,400 units at a price of Euro 500 per unit. The cost of imported components is S$ 800 per
unit. The fixed cost and other variables cost per unit are Rs. 1,000 and Rs. 1,500 respectively. The cash flows in
Foreign currencies are due in six months. The current exchange rates are as follows:
/Euro 51.50/55
/S$ 27.20/25
After six months the exchange rates turn out as follows:
/Euro 52.00/05
/S$ 27.70/75
(1) You are required to calculate loss/gain due to transaction exposure.
(2) Based on the following additional information calculate the loss/gain due to transaction and operating exposure if the
contracted price of air conditioners is Rs.25,000 :
(i) the current exchange rate changes to
/Euro 51.75/80
/S$ 27.10/15
(ii) Price elasticity of demand is estimated to be 1.5
(iii) Payments and receipts are to be settled at the end of six months.

Swap point/Forward margin/Forward Spot differential.


The difference between forward rate and spot rate is known as swap points/forward margin (not exchange margin)/forward
premium (forward premium “in value” not in “%”. If it is in “%” then do as discussed earlier)
How to calculate forward rate using swap point/Forward margin?
 When we are given the Spot rate with forward margin / forward premium / forward swap / forward points for
buying rate or selling rate (Bid or Ask),
 Check whether the forward margin for buying or selling is in increasing order OR in decreasing order, to find the
forward rate.
Swap Point

Add to spot rate to arrive forward rate. Deduct from spot rate to arrive forward rate.

Example 1: Spot Rate $1 = 40.4800 / 4900 Example1: Spot rate $1 = 40. 4800 / 4900
Forward Premium .1100 / .1300 (increasing Forward premium .1255 / .1200 (decreasing
trend) trend)

Solution: Forward Rate: 40.4800 / 40.4900 Solution: Forward Rate: 40.4800 / 40.4900
+ .1100 + .1300 (-) .1255 (-) .1200
40.5900 40.6200 40.3545 40.3700
i.e. Forward rate: $ 1 = 40.5900 / 40.6200 i.e. Forward rate: $ 1 = 40.3545 / 40.3700

Example 2: Spot Rate $1 =  52.60 / 70 Example 2: Spot rate $1 = 52.60 / 70


3 Month Forward 20 / 70 (increasing trend) 3 Month Forward 30 / 25 (decreasing trend)

Solution: Forward Rate: 52.60 / 52.70 Solution: Forward Rate: 52.60 / 52.70
+ . 20 + . 70 (-) .30 (-) .25
52.80 53.40 52.30 52.45
i.e. Forward rate: $ 1 = 52.80 / 53.40 i.e. Forward rate: $ 1 = 52.30 / 52.45

Notes for example-2: The point “7 ” given in spot rate is not swap point because it is not the difference in
forward rate and spot rate. It is simply the quotation method as we quote telephone No 24321152/53 it means we
are quoting two telephone no. (1) 24321152 and (2) 24321153. However point given in 3 month forward is not
quotation method like telephone no. it is difference in forward rate and spot rate.

www.fmguru.org CA. Nagendra P a g e | 2.13


Strategic Financial Management

Exchange margin
Exchange margin is the extra amount or percentage charged by the bank over and above the rate quoted by bank.
How to calculate Exchange rate (for both spot rate and forward rate) using exchange margin?
When we are given the Spot rate / forward rate with margin for buying rate and margin for selling rate then effective
rate will be calculated as:

Deduct margin from buying rate to get desired Add margin to selling rate to get desired exchange
exchange rate. Hence, rate. Hence,
Actual buying rate = Bid rate (1-exchange margin) Actual selling rate = Ask rate (1+exchange margin)

Example:

Example: Given rate $1 = . 40.4800 / 40.4900 Example: Given rate $1 = . 40.8300 / 40.8650
Margin 0.08% for Buying rate Margin .25% for selling rate

Solution: Solution:
Desired buying rate $1=40.4800 (1-0.0008) Desired selling rate $1 = 40.8650 (1+0.0025)
= . 40.4476 = . 40.9672

Logic behind addition or deduction: Everyone wants to buy at low rate and wants to sell at high rate. Hence deduct
margin from buying rate to arrive at low rate and add margin to selling rate to arrive at high rate.

General concept question


Question No. - 2A [June-2009-old-6 Marks]
The following 2-way quotes appear in the foreign exchange market:
Spot 2-months forward
/US $ 46.00/46.25 47.00/47.50
Required:
(i) How many US dollars should a firm sell to get 25 lakhs after 2 months?
(ii) How many Rupees is the firm required to pay to obtain US $ 2,00,000 in the spot market?
(iii) Assume the firm has US $ 69,000 in current account earning no interest. ROI on Rupee investment is 10% p.a.
should the firm encash the US $ now or 2 months later?
Ans: (i) US $ 53191.489; (ii) 92,50,000; (iii) Better to encash after 2 Month.

Benefit of Early Payment


Question No. - 3A [CWA-June-2006-8 Marks]
Electronics Corp. Ltd. your customers, have imported 5, 000 cartridges at a landed cost in Bombay, of US $ 20
each. They have the choice of paying for the goods immediately or in three months time. They have a clean
overdraft limit with you where 18% p.a. rate of interest is charged. Calculate which of the following methods
would be cheaper to your customer.
(a) Pay in three months time with interest at 15% and cover the exchange risk forward for three months.
(b) Settle now at the current spot rate and pay interest to the overdraft for three months. The rates are as
follows:
Bombay  /$ Spot: 31.25 - 31.55, 3 month swap: 25/35
Ans: (a)  33,09,625; (b)  32,96,975

P a g e | 2.14 CA. Nagendra www.fmguru.org


FOREX

Question No. - 3B [Nov-2011-6 Marks] [CWA-Dec-2003-8 Marks] [As CWA-Dec-2002-8 Marks]


An import house in India has bought goods from Switzerland for CHF 10, 00,000. The exporter has given the
Indian company two options:
Pay immediately the bill for CHF 10,00,000.
Pay after 3 months, with interest @ 5%p.a.
The importer bank charges 14% on overdrafts. If the exchange rates are as follows, what should the company
do?
Spot (/CHF): 30.00/30.50
3 months (/CHF): 31.10/31.60
Ans: Immediate payment =  315,67,500; pay at 3 month =  319,95,000

Question No. - 3C [Study Mat.]


U.S. Co., purchased , , Mark’s worth of machines from a firm in Dortmund, Germany the value of the dollar
in terms of the mark has been decreasing. The firm in Dortmund offers 2/10, net 90 terms. The spot rate for one
mark is dollar 0.55; the 90 days forward rate is $0.56.
(a) Compute the $ cost of paying the account within 10 days.
(b) Compute the $ cost of buying a forward contract to liquidate the account in 90 days.
(c) The differential between part (a) and part (b) is the result of the time value of money (the discount for
prepayment) and protection from currency value fluctuation. Determine the magnitude of these
components.
Ans: (a) $ 53,900; (b) $56,000; (c) Time value of money = $1120; protection from devaluation = $980

Question No. - 3D
A firm is contemplating import of a consignment from the USA for a value of US dollars 10,000. The firm requires
90 days to make payment. The supplier has offered 60 days interest free credit and is willing to offer additional
30 days credit at an interest rate of 6% p.a. The bankers of the firm offer a short loan for 30 days at 9% p.a. The
bankers quotation for foreign exchange is:
Spot USD 1 =  46.00
60 days forward 1 USD =  46.20
90 days forward 1 USD = 46.35
You are required to advice the firm as to whether it should pay the supplier in 60 days or avail the supplier’s
offer of 90 days credit. Show your calculations.
Ans: pay in 60 days =  465,465; pay in 90 days = 465,818

Cross Rates of Foreign Exchange


The cross rate is the currency exchange rate between two currencies, where neither of the currencies are of the
country in which the exchange rate is quoted.
Example of cross rate:
Exchange rate: £ 1 = € 1.21 in India.  Exchange rate: € 1 = ¥ 106 in India.
In both rate neither of the currencies is of India (i.e. ) hence it is cross rate, in India.

 How to calculate required exchange rate using cross rate?


Suppose an Indian importer have to make payment in ‘¥’ currency but ‘¥’ currency quote is not available directly
with.

www.fmguru.org CA. Nagendra P a g e | 2.15


Strategic Financial Management

Option-I [If quoted rate is one way quote]


Rate quoted by Indian bank is: (i) £ 1 = $ 1.55 (ii) £ 1 = 72 (iii) € = $ .27 (iv) € = ¥ 6
What is the applicable rate to Indian importer for payment in ¥ ?
We can calculate required rate /¥ using following relationship:
  £ €
= = = 0.5565
¥ £ € ¥

 Hence, required rate: ¥1 =  0.5565

Option-II [If quoted rate is two way quote]


Rate quoted by bank is: (i) £ 1 = ¥ 130 – 131 (ii) £ 1 = 72 – 72.5
OR, ¥1 = £ – OR, 1 = £ –

Calculate required rate using following relationship:


  £  £
= – = – = 0.5496 – 0.5577
¥ £ ¥ £ ¥

for Bid for Ask

 Hence, required rate: ¥1 = 0.5496 – 0.5577

Question No. - 4A [RTP-Nov-2009-old]


You sold € million value spot to your customer at € = 54.60 and covered yourself in Singapore market on the
same day when the exchange rate was as under:
Spot US$ = € .7373/ .7375
Local inter bank US$ 1 = 40.1850/40.1950
(Brokerage paid 2,000)
Calculate the cross rate nearest to the fourth decimal and ascertain profit or loss in the transaction to the nearest
rupee.
Ans: Cross rate =  54.5165; Profit = 81,500

Question No. - 4B [Nov.-1998-8marks] [Study Mat.] [As RTP-June-2009-Old]


X Ltd. an Indian company has an export exposure of 10 million (100 lakhs) yen, value September end. Yen is not
directly quoted against Rupee. The current spot rates are USD/INR = 41.79 and USD/JPY = 129.75.
It is estimated that Yen will depreciate to 144 level and Rupee to depreciate against dollar to 43.
Forward rate for September 1998 USD/Yen = 137.35 and USD/INR = 42.89.
You are required to:
(i) to calculate the expected loss if hedging is not done. How the position will change with company taking
forward cover?
(ii) If the spot rate on 30th September, 1998 was eventually USD/Yen = 137.85 and USD/INR = 42.78, is the
decision to take forward cover justified?
Ans: (i) Expected loss = 2.347 Lacs; if forward cover taken loss can be reduced to 0.981 lacs,
(ii)Receipt under Forward cover = 31.227 Lacs; decision to purchase forward cover is justified
Question No. - 4C [Nov.-2005-old-4marks] [As RTP-June-2009-old] [CS-June-09-4 Marks]
You sold Hong Kong Dollar 1,00,00,000 value spot to your customer at  5.70 & covered yourself in London
market on the same day, when the exchange rates were
US$ 1 = H.K.$ 7.5880 - 7.5920
Local inter bank market rates for US$ were
Spot US$ 1 =  42.70 - 42.85
Calculate cover rate & ascertain the profit or loss in the transaction ignores brokerage.
Ans: Cover rate HK$ 1 = 5.62434 – 5.647074; Gain =  5,29,260;

P a g e | 2.16 CA. Nagendra www.fmguru.org


FOREX

Question No. - 4D [CWA-Dec-2002-4 Marks]


Your bank wants to calculate Rupee. TT selling rate of exchange for DM since a deposit of DM 1,00,000 in a
Foreign Currency Non Resident A/C has matured, when:
EURO 1 = DM 1.95583 (locked in rate)
EURO 1 = US$ 1.02338/43
US $ 1 =  48.51/53
What is the Rupee TT selling rate for DM currency?
Ans: Rupee TT selling rate = 25.3944

Question No. - 4E [Nov-2011-5 Marks] [May-2005-old-8marks] [As Study Mat.]


On January 28, 2005 an importer customer requested a bank to remit Singapore Dollar (SGD) 25,00,000 under an
irrevocable LC. However, due to bank strikes, the bank could effect the remittance only on February 4, 2005.
The interbank market rates were as follows:
January, 28 February 4
Bombay US$1 =  45.85/45.90 45.91/45.97
London Pound 1 = US$ 1.7840/1.7850 1.7765/1.7775
Pound 1 = SGD 3.1575/3.1590 3.1380/3.1390
The bank wishes to retain an exchange margin of 0.125%. How much does the customer stand to gain or lose
due to the delay?
(Calculate rate in multiples of .0001)
Ans: Loss to customer due to strike = 2,28,250

Question No. -4F [RTP-Nov-2010-Old]


Somu Electronics imported goods from japan on july 1st 2009, of ¥ 1 Million, to be paid on 31st, December 2009. Mr. X,
the treasury manager collected the following exchange rates on July 01, 2009 from the bank.
Delhi /$ Spot 45.86/88
6 months forward 46.00/03
Tokyo ¥/$ Spot 108/108.50
6 month forward 110/110.60
In spite of fact that the forward quotation for ¥ was available through cross currency rates, Mr. X, the treasury
manager purchased spot $ and converted $ into ¥ in Tokyo using 6 months forward rate.
However, on 31st December, 2009 /$ spot rate turned out to be 46.24/26.
You are required to calculate the loss or gain in the strategy adopted by Mr. X by comparing the notional cash flow
involved in the forward cover for yen with the actual cash flow of the transaction.
Ans: Loss = 2,091 [Hint: Cash flow of forward purchasing ¥ = 418454.50;
Cash flow of forward purchasing $ in spot market and converting into ¥ 420545.50]

www.fmguru.org CA. Nagendra P a g e | 2.17


Strategic Financial Management

Hedging through Money (cash) Market Operation:


A. Amount Receivable in foreign currency:
Ind Co. cannot receive payment today.
It cannot use forward rate.
It has to hedge transaction using spot rate but it will receive
fund after 3M.

Export to USA

1 Invoice for
$ 10,000/-
Indian Company Credit period 3M USA Company
Inflow at 3 Month Outflow at 3 Month

4 3 $ 5
2
 3M
 today  $

Indian B – 1 Indian B – 2 USA Bank


Rate: 10-12% SR: 1$=40-41 Rate; 5%-6%

2 Borrow less amount today from the USA bank for 3 month at borrowing rate of USA.
= $ 10,000 / (1+0.06 X 3/12) = $ 9,852

3 Convert $ 9,852 into ‘Rupee’ @ spot rate.


$9,852 =  9,852 X 40 =  3,94,080

4 Invest  3,94,080 in India for 3 month at Deposit rate of India.


Deposit amount after 3 month =  3,94,080 X (1+0.10X3/12) =  4,03,932

Borrowing amount with interest will become Invoice amount at 3 month time which will pay by USA
5
Company to USA Bank.
Hence, Amount receivable under money market operation at 3 month time is equal to  4,03,932/-

Concept:
Why borrowed money from foreign bank to be deposit in home country bank? Why not keep with him
today?
 We know comparison cannot be done between rickshaw wale and chartered accountant. Similarly, today
payment cannot be comparable with Future payment.
If we do not deposit money in home country bank today, then the inflow become today’s time and it is not
comparable with forward contract or with other hedging option. Hence deposit today in home country bank
by taking loan in foreign currency.

P a g e | 2.18 CA. Nagendra www.fmguru.org


FOREX

B. Amount payable in foreign currency:


Ind Co. not makes payment today.
It cannot use forward rate.
It has to hedge using spot rate but make payment after 3M.

Import from USA

1 Invoice for
$ 10,000/-
Indian Company Credit period 3M USA Company
Inflow at 3 Month Outflow at 3 Month

2 3 $
5
4
 3M
 today  $

Indian B – 1 Indian B – 2 USA Bank


Rate: 10-12% SR: 1$=40-41 Rate; 5%-6%

2 Borrow money from home country. (How much money we will borrow?)

3 Convert  into $ at spot rate. (Which amount we will convert?)

4 Deposit $ into foreign bank at deposit rate of foreing country

We cannot calculate the required amount if we follow above sequence. Hence do back calculation.
Deposit less amount today in the USA bank for 3 month at deposit rate of USA.
4
= $ 10,000/ (1 + 0.05 X 3/12) = $ 9,876
Convert $ 9,876 into ‘Rupee’ @ spot rate.
3
$ 9,876 =  9,876 X 41 =  4,04,916

2 Borrow  4,04,916 in India for 3 month at borrowing rate of India.


Borrowing amount after 3 month =  4,04,916 X (1+0.12X3/12) =  4,17,063

Deposit amount with interest will become Invoice amount at 3 month time which will receive by USA
5
company from USA Bank.

Hence, Amount payable under money market operation at 3 month time is equal to  4,17,063/-

Concept: Why borrowed money from home country bank to be deposit in foreign bank? Why not from own pocket
at today time?
If we not borrow money from home country bank, then the outflow become today’s time and it is not
comparable with forward contract or with other hedging option. Hence deposit today by taking loan.

www.fmguru.org CA. Nagendra P a g e | 2.19


Strategic Financial Management

Hedging through Forward market & Money Market Operation


Question No. - 5A [Study Mat.] [As CWA-June-2006-8 Mks]
The finance director of P Ltd., has been studying exchange rates and interest rates relevant to India and USA. P
Ltd. has purchased goods from the US Co. at a cost of $51 lakhs payable in dollars in three months time. In order
to maintain profit margins the finance director wishes to adopt, if possible, a risk -free strategy that will ensure
that the cost of the goods to P Ltd. is no more than  22 Crores.
Exchange rates ( /Dollars)
Spot 40-42
1 month forward 41-43
3 months forward 42-45
Interest rate (available to P Ltd):
India USA
Deposit rate (%) Borrowing rate (%) Deposit rate (%) Borrowing rate(%)
1 month 13 15 7 10
3months 13 16 8 11
Calculate whether it is possible for P .Ltd to achieve a cost directly associated with this transaction of no more
than  22 Crores by means of a forward hedge, or money market hedge.
Ans: payment under forward hedge =  22.95 crore;
Payment under Money market hedge =  21.84 crore

Question No.-5B [Nov-2008-New-6 Marks] [Nov-2009-old-7 Marks] [CS-June-08-12 Marks] [RTP-Nov-2009]


An exporter is a UK based company. Invoice amount is $3,50,000. Credit period is three months. Exchange rates in
London are:
Spot Rate ($/£) 1.5865 – 1.5905
3-month Forward Rate ($/£) 1.6100 – 1.6140
Rates of interest in Money Market:
Deposit Loan
$ 7% 9%
£ 5% 8%
Compute and show how a money market hedge can be put in place. Compare and contrast the outcome with a forward
contract.
Ans: Receipt under money Market hedge = £ 2, 17,902;
Receipt under forward hedge = £ 2, 16,852

Question No. - 5C
F Ltd. is a medium-sized UK company with export and import trade with the USA. The following transaction are
due within the next six months.
Sale of finished goods, cash receipt due in three months $1,97,000
Purchase of finished goods cash payment due in six months $2,93,000
Exchange rates (London Market)
$/£
Spot £ 1 = $ 1.7106-1.7140

Three months forward £ 1 = $ 1.7024-1.7063

Six months forward £ 1 = $ 1.6967-1.7006

Calculate the net sterling pounds receipts and payments that F Ltd. might expect for both its three and six
months transactions if the company hedges foreign exchange risk on (1) forward foreign exchange market (2) on
money market operation basis. You may assume following interest rates:

P a g e | 2.20 CA. Nagendra www.fmguru.org


FOREX

Borrowing Lending
Pound 12.50%p.a. 9.50% p.a.
$ 9%p.a. 6% p.a.
Ans: Receipt Forward =£ 1,15,454.49; Receipt money market = £ 1,15,076.33;
Payment Forward = £ 1,72,688.16; Payment money market = £ 1,76,689.55

Question No. - 5D [Study Mat.] [As RTP-Nov-2008-old] [Must Revise]


On 1st March 2009, the B Ltd. bought from a foreign firm electronic equipment that will require the payment of
LC 9, 00, 000 on 31st May 2009. The spot rate on 1st March 2009, is LC 10 per dollar; the expected future spot
rate is LC 8 per dollar; and the 90 days forward rate is LC 9 per dollar.
The US interest rate is 12%, and the foreign interest rate is 8%. The tax rate for both countries is 40%. The B Ltd.
is considering three alternatives to deal with the risk of exchange rate fluctuations.
(a) To enter the forward market to buy LC 9,00,000 at the 90 days forward rate in effect on 31st May 2009.
(b) To borrow an amount in dollars to buy the LC at the current spot rate. This money is to be invested in
government securities of the foreign country; with the interest income, it will equal LC 9, 00,000 on 31st
May 2009.
(c) To wait until 31st May 2009, and buy LCs at whatever spot rate prevails at that time.
Which alternatives should the B Ltd. follow in order to minimize its cost of meeting the future payment in
LCs? Explain.
Ans: (a) Forward Mkt. = $ 96, ; (b) Money Mkt. = $ 9 ,534; (c) wait until … = $ , 3,5

Question No. - 5E [Nov-2008-New-6 Marks] [RTP-Nov-2009]


An Indian exporting firm, Rohit and Bros., would be cover itself against a likely depreciation of pound sterling. The
following data is given:
Receivables of Rohit and Bros : £500,000
Spot rate : 56,00/£
Payment date : 3-months
3 months interest rate : India: 12 per cent per annum
: UK: 5 per cent per annum
What should the exporter do?
Ans: Amount receivable = 284,83,941; Net gain = 4,83,941

Question No. - 5F
Best of Luck Ltd., London will have make a payment of $ 3,64,897 in six month’s time. It is currently st October.
The company is considering the various choice it has in order to hedge its transaction exposure.
Exchange rates
Spot rate $1.5617-1.5773
six-month forward rate $1.5455-1.5609

Money market rates: Borrow(%) Deposit(%)


US 6 4.5
UK 7 5.5

By making the appropriate calculations decide which of the following hedging alternatives is the most attractive
to Best of Luck Ltd. (a) Forward market, (b) Cash (Money) market.
Ans: (a) Pmt. under Forward Mkt. = £ 236102.88; Pmt. under Money Mkt. = £ 2,36,510.11;

www.fmguru.org CA. Nagendra P a g e | 2.21


Strategic Financial Management

Question No. - 5G [CWA-June-2004-8 Mks]


MN, a UK company, has a substantial portfolio of its trade with American and German companies. It has recently
invoiced a US customer the sum of $ 50 ,00,000, receivable in one year’s time. MN finance director is considering
two methods of hedging the exchange risk:
Method 1: Borrowing present value of $ 5 million now for one year, converting the amount into sterling, and
repaying the loan out of eventual receipts.
Method 2: Entering into a 12-month forward exchange contract with the company’s bank to sell the $5 million.
The spot rate of exchange is £ 1 = US $ 1.6355,
The 12-month forward rate exchange is £ 1 = US $ 1.6125,
Interest rates for 12 months are USA 3.5%; UK 4%.
You are required to calculate the net proceeds in sterling under both methods and advise the company.
Ans: Rcpt. under Method 1 = £ 3071937.99; Rcpt. Under Method 2 = £ 3100775.19

Question No. - 5H
Expo is an importer/exporter of textile machinery. It is based in the UK but trades extensively with countries
throughout Europe. It has a small subsidiary based in Germany. The company is about to invoice a customer in
Germany DM 7, 5 , payable in three month’s time. Expo’s treasurer is considering two me thods of hedging
the exchange risk;
Method 1: Borrow DM 7, 50,000 for three months, convert the loan into sterling and repay the loan out of
eventual receipts.
Method 2: enter into a three month forward exchange contract with the company’s bank to sell DM 7,5 , .
The spot rate of exchange is DM 2.3834 to the £ 1.
The three month forward rate of exchange is DM 2.3688 to the £ 1.
Annual interest rates for three months borrowing are: Germany 3%, UK 6%.
Which of the two methods are the most financially advantageous for Expo?
Ans: Rcpt. Under method 1 = £ 317022.04; under Method 2 = £ 316,616.
[In absence of deposit rate we use borrowing rate as deposit rate.]

Question No.- 5I [RTP-May-2010-New/Old]


Wenden Co is a Dutch-based company which has the following expected transactions.
One month: Expected receipt of £2,40,000
One month: Expected payment of £1,40,000
Three months: Expected receipts of £3,00,000
The finance manager has collected the following information:
Spot rate (£ per €) : 1.7820 ± 0.0002
One month forward rate (£ per €) : 1.7829 ± 0.0003
Three months forward rate (£ per €) : 1.7846 ± 0.0004
Money market rates for Wenden Co:
Borrowing Deposit
One year Euro interest rate: 4.9% 4.6
One year Sterling interest rate: 5.4% 5.1
Assume that it is now 1 April.
Required:
(a) Calculate the expected Euro receipts in one month and in three months using the forward market.
(b) Calculate the expected Euro receipts in three months using a money-market hedge and recommend whether a
forward market hedge or a money market hedge should be used.
Ans: (a) Expected euro receipt: in 1 m = €56, 79, in 3 m = € ,68, 67.
(b) Expected euro receipt in 3 m = € 67,999.

P a g e | 2.22 CA. Nagendra www.fmguru.org


FOREX

Question No. - 5J
LMN exports its products throughout the world. It has today received from a new customer in Uganda an order
worth £ , 5 , at today’s spot rate. The order is to be paid in the importer’s currency. Terms of trade are 6
days’ credit. No discount is offered for early payment.
Foreign exchange rates (mid rates)

US$/£ Uganda shillings/£


Spot 1.492 1,700
1 month forward 1.4898 N/A
2 month forward 1.4886
3 month forward 1.4873

Money market rates (% per annum)


Deposit Borrowing
UK bank 5 8
Uganda bank 15 N/A
US domestic bank 3 6
LMN converts all foreign currency receipts into sterling immediately on receipt. Wherever possible, the company
uses forward exchange contracts to hedge its currency risks.
In view of the lack of forward markets in Uganda, the Ugandan customer has offered to pay $US 2, 25,000 to LMN
in three months’ time, instead of Ugandan shillings in 6 days. The customer is able to do this as a result of his
government’s new economic liberalization policies.

You are required: to calculate the expected sterling receipts from the Ugandan customer, assuming its offer of
payment in US$ is accepted. Assume LMN hedges its risk using
(1) The forward market, or
(2) The money market,
and advise LMN on which method is most advantageous; to advise LMN on whether the Ugandan customer’s
offer of payment in US$ should be accepted.
Ans: Receipt under1 = £ 1,51,281; Receipt under 2 = £ 1,50,433

Leading/Lagging
Leading:
Lead payment is a payment as on today for a transaction forgetting the credit period given by supplier.
Importer makes the lead payment when he feels that the loss due to currency fluctuation will become higher
than that of the benefit which he will get due to time value of money.

Notes: We know comparison can never be done between rickshaw wale and chartered accountant. Similarly,
Lead payment can never be comparable with Forward cover because under lead payment outflow become
today and under forward cover outflow become at some future date.
Hence, for making lead payment comparable with forward cover we make payment today taking loan (for the
period equal to forward cover) so that the outflow become at the time of repayment of loan.

www.fmguru.org CA. Nagendra P a g e | 2.23


Strategic Financial Management

Lagging:
Lagging means delaying the payment beyond the due date allowed by supplier.

Example: Indian importer has to make payment for $ 5,000 at 3 month time to US supplier. If payment is
delayed, supplier charges interest @ 6% p.a.
3 month forward rate: 1$ = 45.30- 45.36
6 month forward rate: 1$ =  43.20-43.50
Indian company is considering the lagging if it is beneficial. Cost of capital of Indian company is 12%.

1. If Indian importer make payments at due date i.e. at 3 month time:


Outflow in Rupee at 3 month = 5,000 45.36 = 2,26,800

2. In Indian importer make payment at 6 month time (i.e. lagging):


Outflow in Dollar at 6 month= 5,000 = $ 5,075 [charge int. for 3M only)
Outflow in Rupee at 6 month= 5,075 43.50 = 2,20,762.5

0 Year 3 month 6 month

2,20,762.5

= 2,14,332

Outflow under lagging will be 214332 and under normal credit period will be 2,26,800. Hence lagging is
beneficial for importer.

Question No.- 6A [RTP-May-2010-New/Old]


CQS plc is a UK company that sells goods solely within UK. CQS plc has recently tried a foreign supplier in Netherland
for the first time and need to pay €25 , to the supplier in six months’ time. You as financial manager are
concerned that the cost of these supplies may rise in Pound Sterling terms and has decided to hedge the currency risk
of this account payable. The following information has been provided by the company’s bank:
Spot rate (€ per £): 1·998 ± 0·002
Six months forward rate (€ per £): 1·979 ± 0·004
Money market rates available to CQS plc:
Borrowing Deposit
One year Pound Sterling interest rates: 6·1% 5·4%
One year Euro interest rates: 4·0% 3·5%
Assuming CQS plc has no surplus cash at the present time you are required to evaluate whether a money market
hedge, a forward market hedge or a lead payment should be used to hedge the foreign account payable.
Ans: (a) Money market: £ 1,26,850; Forward market: £ 1,26,582; Lead payment: £1,29,071.

Question No.-6B [May-2012-8 Marks]


NP and Co. has imported goods for US $ 7, 00,000. The amount is payable after three months The company has also
exported goods for US $ 4,50,000 and this amount is receivable in two month. For receivable amount a forward
contract is already taken at  48.90.
The market rates for  and Dollar areas under:
Spot 48.50/70
Two months 25/30 point
Three month 40/45 point

P a g e | 2.24 CA. Nagendra www.fmguru.org


FOREX

The company wants to cover the risk and it has two options as under:
(i) To cover payables in the forward market and
(ii) To lag the receivables by one month and cover the risk only for net amount. No interest for delaying the
receivables is earned.
Evaluate both the options if the cost of Rupee Fund is 12% .Which option is preferable.

Exchange rate determination Theory

Exchange Rate determination theory

Interest rate Parity theory Purchasing power parity International Fishers effect
(IRTP) theory (PPPT) (IFE)

It establishes relationship It establishes relationship It establishes relationship


between FR (Forward Rate), SR (Spot between ER (Expected rate), SR between ER (Expected rate), SR
rate) & Interest rate of two (Spot rate) & Inflation rate of two (Spot rate) & interest rate of two

countries countries. countries.


Relationship Relationship Relationship
FR (/$) = SR (/$) ER (/$) = SR (/$) ER (/$) = SR (/$)
  

Where, Where, Where,


PIR = Periodic Interest rate. PIR = Periodic Inflation rate. PIR = Periodic Interest rate.

Notes:
1. If SR, FR and two countries interest rates are based on IRPT, then Arbitrage gain is not possible.
2. The currency having high interest rate would be at discount and currency having low interest rate would be
at premium in future.
3. The premium/discount on one currency would be approximately equal to the interest rate differential.

Relationship in interest rate as per Fishers [Fishers formula]

Conversion value) become worthless. In other word it is Floor value of bond

Risk Free Rate (Money Rate)-High


Inflation

Real Interest Rate-Low

Relationship:
(1 + Real rate) (1+ Inflation rate) = (1+ Risk free rate)

www.fmguru.org CA. Nagendra P a g e | 2.25


Strategic Financial Management

Interest Rate Parity Theorem (IRPT)


Question No. - 7A (May-2004-old-9marks)
The United States Dollar is selling in India at  45.50. If the interest rate for a 6-months borrowing in India is
8% per annum and the corresponding rate in USA is 2%.
(i) do you expect United States Dollar to be at a premium or at discount in the Indian forward market;
(ii) what is the expected 6-months forward rate for United States Dollar in India; and
(iii) What is the rate of forward premium or discount?
Ans: (i) Dollar will be at premium; (ii) Forward rate (/$) =46.85;
(iii) Forward Premium = 5.934%

Question No. - 7B [Nov-2002-old-4 Marks]


On 1st April, 3 months interest rate in the US and Germany are 6.5 per cent and 4.5 per cent per annum
respectively. The $/DM spot rate is 0.6560. What would be the forward rate for DM for delivery on 30th June ?
Ans: 3 Month Forward rate: DM 1 = $ 0.6592

Question No. - 7C
The spot Danish Krone rate is $ 0.15986 and the three month forward rate is $ 0.1590. The three month treasury
bill rate in the United States is 6.25% p.a. and in Denmark 7.50% p.a.
(a) Calculate forward premium or discount on Danish Krone
(b) Are the forward rates and interest rate in equilibrium?
(c) Work out the forward rate if the forward premium or discount are in equilibrium.
Ans: (a) Annualized Discount = 2.152%; (b) Given DK 1 = $ 0.1590; Calculated, DK 1
= $ 0.15937; Hence not in equilibrium. (c) DK 1 = $ 0.15937.
Question No. - 7D
Spot 1$ =  52.50
Interest Rate ($) = 8% p.a
Interest Rate () = 10% p.a.

(i) Estimate six months forward rate as per IRPT.


(ii) Calculate rate of discount/premium of dollar on the basis of forward and spot rates.
(iii) Calculate rate of discount/premium of dollar on the basis interest rates.
Ans: (i) 6 month forward Rate $1 =  53.0048;
(ii) Annualized Premium = 1.923% (iii) A. Premium = 1.923%

Question No. - 7E
The financial press recently listed the following information about two currencies, the Westland dollar ($W) and
the Eastland mark (EM).
Spot rates: 2.0725 EM / $W
0.4825 $W / EM
90 day rates: 2.0687 EM / $W
0.4834 $W / EM

Westland prime interest rate on the same day was 9.5%. (Note: - Use a 365 day year.)
Requirements:
(i) Explain what is implied about the Eastland interest rate.
(ii) Calculate and comment on the Eastland interest rate if the forward exchange rate was 0.4795 $W/EM.
(iii) Calculate and comment on the 90 day forward rate on EM / $W if the Eastland interest rate was 8%.
Ans: (i) Eastland interest rate = 8.72%; (ii) Eastland interest rate = 12.09%; (iii) $W 1 = EM 2.065

P a g e | 2.26 CA. Nagendra www.fmguru.org


FOREX

Question No. - 7F [Nov.-2000-old-8marks)


The following table shows interest rates for the United States dollar and French francs. The spot exchange rate
is 7.05 francs per dollars. Complete the missing entries:
3 Months 6 Months 1 Year
Dollar interest rate
11½% 12¼% ?
(annually compounded)

Franc interest rate


19½ ? 20%
(annually compounded)
Forward franc per dollar ? ? 7.5200
Forward discount per franc
? 6.3% ?
per cent per year

Ans: 3 Month: FR = 7.17; FD = 6.7%; 6 Month: Franc IR = 19.8%; FR = 7.28; 1 Year: Dollar IR = 12.5%, FD = 6.25%

Question No. - 7G [May-2001-old-10marks]


Shoe Company sells to a wholesaler in Germany. The purchase price of a shipment is 50,000 deutsche marks
with term of 90 days. Upon payment, Shoe Company will convert the DM to dollars. The present spot rate for
DM per dollar is 1.71, whereas the 90-day forward rate is 1.70.
You are required to calculate and explain:
(i) If Shoe Company were to hedge its foreign-exchange risk, what would it do? What transactions are
necessary?
(ii) Is the deutsche mark at a forward premium or at a forward discount?
(iii) What is the implied differential in interest rates between the two countries?
(Use interest-rate parity assumption).
Ans: (i) Hedging through Forward Market, Money Market Operation or currency option & Futures;
(ii)Forward Premium on DM = 2.353%; (iii) interest rate differential = 2.353%

Question No. - 7H
You are given the following interest rates.
 $
3-month 15% 6%
6-month 14.5% 5.5%
9-month 14.0% 5.0%
The 3-month forward rate is  36/$. Calculate the 3-month forward rate 6-months from now.
Ans: Forward rate = 37.51

Purchasing Power Parity OR Inflation rates parity


Question No. - 8A
Sampras Tennis Rackets Cost UK P 100 in the UK and 150 in the US. The current exchange rate is UK P 1= $ 1.50.
Explain what happens if inflation, which is presently 0% in both the UK and the US, increases to 10% in the US.
Ans: Forward Rate £ 1 = $ 1.65
Question No. - 8B
You are given the following information:
Spot rate : DM 1.50 / $
3-month forward rate : DM 1.51 / $
The inflation rate in Germany is 4%. Calculate the inflation rate in the USA assuming that Purchasing Power
Parity holds goods even in the short run.
Ans: Annualized Inflation Rate = 1.32%

www.fmguru.org CA. Nagendra P a g e | 2.27


Strategic Financial Management

Question No. - 8C
An American based FII is looking to invest US$ 10 million in an emerging market. After a careful analysis of
future prospects, India and Malaysia are shortlisted. For the next year, which is also the holding period for the
FII, expected rates of return are 20% and 16% in India and Malaysian markets respectively. Withholding tax
rates applicable on the returns earned are 20% in India and 10% in Malaysia. Other information available with
the FII includes
Exchange rates:
/$ spot 43.50/43.60
MS/$ spot 3.80/3.82
Expected inflation for the next year:
India 4.0%
Malaysia 6.0%
US 2.0%
Assuming that the PPP holds good, where should the FII invest?
Ans: If invest in India, Net gain = $ 1.352; If invest in Malaysia, Net gain = .95 mill.

Question No.-8D [Nov-2008-New-(4 +4) Marks] [part(i)-May-2010-New-4 Marks] [RTP-Nov-2009/May-2010]


(i) The rate of inflation in USA is likely to be 3% per annum and in India it is likely to be 6.5%. The current spot rate
of US $ in India is 43.40. Find the expected rate of US $ in India after one year and 3 years from now using
purchasing power parity theory.
(ii) On April 1, 3 months interest rate in the UK £ and US $ are 7.5% and 3.5% per annum respectively. The UK £/US $
spot rate is 0.7570. What would be the forward rate for US $ for delivery on 30th June?
Ans: (i) Expected rate after 1 Y =  44.8751; after 3 Y =  47.9762; (ii) FR 1 US $ = UK £ 0.7645

Arbitrage
 Arbitrage is an act to earn risk free profit. The act may be:
(i) Sale at high rate & purchase at low rate; (ii) borrowing at low rate & investing at high rate
 Arbitrage is not possible in Forex market if ‘Exchange rates’ and ‘interest rates’ are based on IRPT.

Arbitrage

Geographical Arbitrage Cover Interest Arbitrage


Situation: Situation:
Information of different rate (either one way Information of FR, SR and interest rate of two
quote or two way quote) for various markets countries are given.
are given.
How to Calculate Arbitrage? How to Calculate Arbitrage?
 See Subsequent Page  See Subsequence Page

When arbitrage is possible? When arbitrage is possible?


Arbitrage is possible if there are mismatch Arbitrage is possible if there are mismatch in FR,
in cross rates of various markets. SR and Interest rates.
Note: Note:
- In one way quote, arbitrage gain is possible - We can use short cut to find out the high rate and
from only one route. If gain in one rout, low rate if given quote is single quote.
then there must be loss in another route. - If given rate is two way quote then check for
- In two way quote there may be loss in both arbitrage possibility for all the routs in trial and
routes. But gain is not possible form both error method.
route.

P a g e | 2.28 CA. Nagendra www.fmguru.org


FOREX

Geographical Arbitrage (Calculation of Arbitrage Gain)


Condition:
There must be two quotes available for each currency
Example: Market-1: $1 = 50.00/50.10
Market-2:  1 = ¥2.00/2.20 Here all currencies are quoted two times.
Market-3: £ 1 = ¥ 140/142 Under this condition only arbitrage is possible
Market-4: $1 = £ 0.67/0.70

Assume arbitrageur has cash in any currency (Say arbitrageur has $1,000)
Here two Routes are possible:

Route-1:

$  ¥ £ $
Gain
$ 1,000 Mkt-1: Mkt-2: Mkt-3: Mkt-4: If > $ 1,000
Sale $ & Sale  & Sale ¥ & Sale £ &
If < $ 1,000 Loss
Buy  Buy ¥ Buy £ Buy $

Route-2:

$ £ ¥  $
Gain
$ 1,000 Mkt-4: Mkt-3: Mkt-2: Mkt-1: If > $ 1,000
Sale $ & Sale  & Sale ¥ & Sale £ &
If < $ 1,000 Loss
Buy  Buy ¥ Buy £ Buy $

Short cut for finding the countries to be invested or to be borrowed when one way quote is given (For Cover
interest arbitrage)

 Calculate Premium/Discount on any currency using following formula


Premium/discount = [Say calculated premium on one currency is 4%]
 Compare this 4% with interest rate differential of two countries and decide the country in which deposit should
be made and country from which borrowing should be made.

Interest rate differential > Premium/Discount Borrow at low rate and deposit at high rate
Interest rate differencial <Premium/Discount Borrow at high rate and deposit at low rate

Example:
Case-I Case-II
Interest rate (India) 10% 7%
Interest rate (USA) 5% 4%

Case-I Case-II
Interest rate differential = 10%-5% = 5% Interest rate differential = 7%-4% = 3%
Premium = 4% Premium = 4%
Here benefit is due to Interest rate. Hence borrow at Here benefit is due to Exchange rate. Hence borrow
low rate (i.e. 5%) from USA and deposit at high rate at High rate (i.e. 7%) from Indian and deposit at Low
(i.e. 10%) from India for Arbitrage. rate (i.e.4%) from USA for Arbitrage.

www.fmguru.org CA. Nagendra P a g e | 2.29


Strategic Financial Management

Geographical Arbitrage
Question No. -9A [Same as Q-14B]
Assuming no transaction costs, suppose £1 = $2.4110 in New York, $1 = FF 3.997 in Paris, and FF 1 = £0.1088 in
London. How would you take profitable advantage if you had £ 50,000?
Ans: Profit = £ 2,424.01

Question No. - 9B [Nov-2008-old-6 Marks] [Same as Q-14A]


Followings are the spot exchange rates quoted at three different forex markets:
USD/INR 48.30 in Mumbai
GBP/INR 77.52 in London
GBP/USD 1.6231 in New York
The arbitrageur has USD1,00,00,000. Assuming that there are no transaction costs, explain whether there is any
arbitrage gain possible from the quoted spot exchange rates.
Ans: Arbitrage gain = USD 1,12,968

Question No. - 9C
Frank furt Spot 1$ = 1.3689 – 1.4150DM
Mumbai Spot 1 DM = 37.5051 – 37.6050 
London Spot 1$ = 50.5020 - 50.6525 
Can you make profit through arbitrage? Assuming Arbitrageur has $1,000
Ans: Arbitrage gain = $ 13.587

Question No. - 9D
Spot rate (Frank furt) 1$ =1.3579-1.3585DM
Spot rate (New York) 1 DM = 0.7149-0.7295$
You have 1,00,000 DM what amount of profit you can make from arbitrage?
Ans: Arbitrage gain = DM 905.55 [if sell DM in frank furt & Purchase back in new York]

Cover interest Arbitrage


Question No. - 10A [May-2006-old-8marks]
Given the following information:
Exchange rate – Canadian dollar 0.665 per DM (spot)
Canadian dollar 0.670 per DM (3 months)
Interest rates – DM 7% p.a.
Canadian Dollar 9% p.a.
What operations would be carried out to take the possible arbitrage gains?
Ans: Arbitrage gain = Can $ 26 [Raise $ loan and Invest in DM; Assumed can $ loan = 10,000]

Question No. - 10B [Nov-2006-old-5marks] [As CWA-Dec-2007-6 Mks]


Spot rat 1 US $ = 48.0123
180 days Forward rate for 1 US $ = 48.8190
Annualised interest rate for 6 months – Rupee = 12%
Annualised interest rate for 6 months – US $ = 8%
Is there any arbitrage possibility? If yes how an arbitrageur can take advantage of the situation, if he is willing to
borrow 40,00,000 or US $83,312.
Ans: Yes, US $ 206.95 or  10,013

P a g e | 2.30 CA. Nagendra www.fmguru.org


FOREX

Question No. - 10C [CWA-Dec-2004-10 Mks]


The annual interest rate is 5% in the United States and 8% in the UK. The spot exchange rate is £/$ = 1.50 and
forward exchange rate, with one year maturity, is £/$ = 1.48. In view of the fact that the arbitrager can borrow $
10, 00,000 at current spot rate what would be the arbitrager profit or loss?
Ans: Arbitrager gain = $ 15,600 (approx)

Question No. - 10D [Study Mat.]


Following are the rates quoted:
/BP 52.60/70 Interest Rates India London
3m forward 20/70 3 months 8% 5%
6m forward 50/75 6months 10% 8%
Verify whether there is any scope for covered interest arbitrage if you borrow rupees.
Ans: Arbitrage not possible either for 3 month period or for 6 month period. [loss 3 month = -558; 6 month = -211]

Question No. - 10E


Consider the following data:
$/£ spot: 1.7500/10
3-month forward: 1.7380/1.7400
3-month dollar: 8.00/8.20%p.a.
3-month Sterling: 10.50/11.00% p.a.
Check whether there is a covered interest arbitrage opportunity.
Ans: Not possible through any route

Question No. -10F [CWA-Dec-2006-8 Mks]


Given the following information:
Spot rate : 46.88/$
3 month forward rate : 47.28/$
3 month interest rate in USA : 7% p.a.
3 month interest rate in India : 9% p.a.
Assuming no transaction cost or taxes exist, what operation would be carried out to take the possible arbitrage gain?
Assume 10 million/$ 10 million borrowings (as the case may be) to explain your answer
Ans: Arbitrage gain = 0.0366 million

Question No. -10G [CWA-June-2005-10 Mks]


Assuming that no transaction sot or taxes exist, do the following data offers arbitrage profit opportunity?
If so, how will the arbitrage transaction be carried out? Assume that arbitrageur can borrow up to $1 million.
Three month interest rate in the United States : 8% per annum
Three month interest rate in Germany : 5% per annum
Current spot exchange rate :€ . 4/$
Three month forward exchange rate :€ . /$
Ans: Arbitrage profit $6,197

www.fmguru.org CA. Nagendra P a g e | 2.31


Strategic Financial Management

Question No. -10H [CWA-Dec-2005-8 Mks]


Consider the following:
Spot rate = $0.75/DM
Forward rate (one year) = $0.77/DM
Interest rate (DM) = 7% p.a.
Interest rate ($) = 9% p.a.
(i) Assuming no transaction cost or taxes exist, do covered arbitrage profits exist in the above situation?
Explain
(ii) Suppose now that transaction costs in the foreign exchange market equal 0.25% per transaction. Do
unexploited covered arbitrage profit opportunities still exist?
Ans: Yes, arbitrage profits 0.85%, (ii) No

Question No. 10i [CS-Dec-2005-15 marks]


Syntex Ltd. has to make a $5 mill payment in three months time. The required amount in dollars is available with
syntax ltd. The management of the company decides to invest them for three months and following information is
available in this context:
The $ deposit rate is 9% pa
The sterling pound deposit rate is 11% pa
The spot exchange rate is $1.82/£
The three month forward rate is $1.80/£
Answer the following questions
(i) Where should the company invest for better returns?
(ii) Assuming that the interest rates and the spot exchange rate remain as above, what forward rate would
yield an equilibrium situation?
(iii) Assuming that the US interest rate and the spot and forward rates remain as above, where should the
company invest if the sterling pound deposit rate were 15% per annum
(iv) With the originally stated spot and forward rates and the same dollar deposit rate, what is the
equilibrium sterling pound deposit rate?

Question No. – 10J [RTP-May-2012]


The risk free rate of interest rate in USA is 8% p.a. and in UK is 5% p.a. The spot exchange rate between US $ and UK £
is 1$ = £ 0.75.
Assuming that interest is compounded on daily basis then at which forward rate of 2 year, there will be no
opportunity for arbitrage.
Further, show how an investor could make risk-less profit, if two year forward price is 1 $ = 0.85 £.
Given e-0.06 = 0.9413 & e-0.16 = 0.852, e0.16 = 1.1735, e-0.1 = 0.9051

Question No. -10K [RTP-May-12]


True Blue Cosmetics Ltd. is an old line producer of cosmetics products made up of herbals. Their products are popular
in India and all over the world but are more popular in Europe.

The company invoice in Indian Rupee when it exports to guard itself against the fluctuation in exchange rate. As the
company is enjoying monopoly position, the buyer normally never objected to such invoices. However, recently, an
order has been received from a whole-seller of France for FFr 80,00,000. The other conditions of the order are as
follows:

(a) The delivery shall be made within 3 months.


(b) The invoice should be FFr.

Since, company is not interested in losing this contract only because of practice of invoicing in Indian Rupee. The
Export Manger Mr. E approached the banker of Company seeking their guidance and further course of action.
The banker provided following information to Mr. E.

P a g e | 2.32 CA. Nagendra www.fmguru.org


FOREX

(a) Spot rate 1 FFr =  6.60


(a) Forward rate (90 days) of 1 FFr = 6.50
(b) Interest rate in India is 9% and in France is 12%.

Mr. E entered in forward contract with banker for 90 days to sell FFr at above mentioned rate.
When the matter comes for consideration before Mr. A, Accounts Manager of company, he approaches you.
You as a Forex consultant are required to comment on:
(i) Whether there is an arbitrage opportunity exists or not.
(ii) Whether the action taken by Mr. E is correct and if bank agrees for negotiation of rate, then at what forward
Rate Company should sell FFr to bank.

Rupee Roll-Over-forward Contracts


Forward rate for the period more than 6 month is not available. In this case rupee role over contract is used to hedge
the future payment or receipt.
When whole amount in foreign currency is receivable or payable at the time more than 6 month.
Amt receivable/Payable
$1,00,000 at 2Y
0Y 6m 1Y 1.5 Y 2Y

1 2 3 4 5

1. Originally enter 6m forward contract @ 6m FR on 0 Y to sell/buy $1,00,000/-


2. At 6m time, cancel forward contract by entering opposite contract (because payment not receivable/payable at 6m ) @ SR on 6
Month.
- Calculate amount receivable/payable due to cancelation.
- Again enter 6m forward contract @ 6m FR on 6m to sell/Buy $ 1,00,000/-

3. At 1Y, cancel forward contract by entering opposite contract (because payment not receivable/payable at 1Y) @ SR on 1 Year.
- Calculate amount receivable/payable due to cancelation.
- Again enter 6m forward contract @ 6m FR on 1 Y to sell/Buy $ 1,00,000/-

4. At 1.5Y, cancel forward contract by entering opposite contract (because payment not receivable/payable at 1.5Y) @ SR on 1.5
Year.
- Calculate amount receivable/payable due to cancelation.
- Again enter 6m forward contract @ 6m FR on 1.5 Y to sell/Buy $ 1,00,000/-

5. At 2Y, sell/Buy $1,00,000 @ committed 6m FR of 1.5Y time.


- Calculate final amount receivable/payable.

Total amount receivable/payable under Rupee roll over:


A. When cost of debt/discount rate is not given:
Amount receivable/payable = Amt at 6m + Amt at 1Y + Amt at 1.5Y + Amt at 2Y.

www.fmguru.org CA. Nagendra P a g e | 2.33


Strategic Financial Management

B. when cost of debt/discount rate is given:

Option – 1:
Total amount receivable/payable under Rupee roll over at future’s money term i.e. after 2Y:
Amt at 6m Amt at 1Y Amt at 1.5Y Amt at 2Y
Future value of receivable/payable = + + +
(1+PIR)3 (1+PIR)2 (1+PIR)1 (1+PIR)0

Option – 2:
Total amount receivable/payable under Rupee roll over at today’s money term:
Amt at 6m Amt at 1Y Amt at 1.5Y Amt at 2Y
Present value of receivable/payable = + + +
(1+PIR) 1 (1+PIR) 2 (1+PIR) 3 (1+PIR) 4
OR
Amt at 6m Amt at 1Y Amt at 1.5Y Amt at 2Y
+ + +
PVIF(R, 1) PVIF(R, 2) PVIF(R, 3) PVIF(R, 4)

Question No. - 11A


A person gets an interest free loan of USD 3,00,000. Repayment is to be done in three equal half yearly
installments. Assume the following rates:
A. Today Six months forward rate 42.00 - 42.50
B. At the end of six months Spot  43.00 - 43.10
Six months forward 43.40 - 43.50
C. At the end of one year Spot  44.00 - 44.10
Six months forward  44.50 - 44.60
At the end of one & half
D. Spot  45.00 - 45.10
year
Find amount he has to pay in rupees in following three cases
(i) No hedging
(ii) Rupee roll over forward
(iii) Three separate forward contracts one today, one after six months and one after one year from today .
Ans: (i)  132,30,000; (ii)  129,10,000; (iii)  130,60,000 [ prefer Rupee roll over forward contract]

Question No. - 11B


An Indian corporate completes a project of value, $1 million in the Middle East on 31/12/96. The contract has
been executed on deferred payment terms and the necessary permission for late realization of export proceeds
has been obtained from the Reserve Bank of India. The company has to bear the currency risk however till
1/1/99 when payment will be realized. When the corporate approaches its bank, it is informed that contracts of
maturity greater than six months cannot be structured. It hence opts for six-month roll over cover.

The following are the spot rates ( /$) and six-month forward rates (,/$) prevailing at the end of each roll over
period. Determine the cash flows and compare the result with a situation when the corporate leaves the
exposure uncovered. Cost of capital for the company is 20%.

P a g e | 2.34 CA. Nagendra www.fmguru.org


FOREX

Date Spot Rate ( / $) Six-month Forward Rate


1/1/97 35.00 35.20
1/7/97 35.15 35.30
1/1/98 35.25 35.35
1/7/98 35.35 35.50
1/1/99 35.45 35.60
Ans: PV of inflows: under rupee roll over =  25,11,400; Under no hedging = 24,21,235

Nostro A/C ; Vostro A/C & LORO Account


Nostro A/C: [Ours Account with You]
Nostro Account is a Current account maintained by a domestic bank/dealer with a foreign bank in foreign currency.
For example, Current Account of SBI Bank (an Indian bank) with Swiss Bank in Swiss Franc (CHF) currency is a Nostro
A/C.

India Switzerland

Indian Bank Swiss Bank


Nostro Account of
Indian bank
Indian Bank opened a Current Account with
Swiss Bank in CHF currency. This Bank account
is known as Nostro Account for Indian.

Vostro A/C: [Yours Account with us]


Vostro A/C is a Current account maintained by a foreign bank with domestic bank in Rupee currency.
For example: Account of Swiss bank in India with SBI in Rupee () Currency.

India Switzerland

Indian Bank Swiss Bank


Vostro Account of
Swiss bank
Swiss Bank opened a Current Account with
Indian Bank in  currency. This Bank account
is called Vostro Account by Indian.

www.fmguru.org CA. Nagendra P a g e | 2.35


Strategic Financial Management

Loro A/C: [Our Account of their money with you]


Loro Account is a Current Account Maintained by one Domestic Bank on behalf of other domestic bank in foreign bank
in foreign currency. In other word Loro Account is a Nostro Account for one bank who opened the bank and Loro
Account for other bank who refers first one account.
For Example: SBI opened Current Account with Swiss bank. If PNB refers that account of SBI for its correspondence,
then it is called Loro Account for PNB and it is Nostro Account for SBI.

India Switzerland
SBI Bank opened a Current Account
with Swiss Bank in CHF currency.
Indian Bank (SBI) Swiss Bank
Nostro Account of
Indian bank

Indian Bank (PNB)


PNB Bank refers Account of SBI for its transaction
then it is called Loro Account for PNB

Maintaining Exchange position and Nostro A/C [Nov-05] [RTP-Nov-08]


General information:
- Currency of Switzerland is Swiss Franc (CHF)
- “Zurich” is a city of Switzerland
- Telegraphic Transfer or Telex Transfer , often abbreviated to TT, is an electronic means of transferring
funds overseas

Exchange position: Exchange position (for CHF currency) of an Indian bank/dealer will affect by:
- Purchase/sale of foreign currency, (the purchase/sale of currency may be spot or forward)
- Issue/cancellation of demand draft,
- Purchase/sale of Bills receivable,
- Remittance of foreign currency [Remember that Indian bank do not remit foreign currency (say CHF) in Nostro account from India. All receipt in
Swiss franc (CHF) and all payment/remittances in CHF will be made through Nostro account maintained with Switzer land bank]

Cash Position (Nostro A/C): Nostro A/C of an Indian bank/dealer maintained with Switzerland bank will affect by:
- Spot Purchase/Sale of foreign currency (CHF) (Forward Purchase/Sale of CHF do not affect Nostro A/C because there is no delivery of
currency as on today)
- Receipt/payment in CHF.
Note: Spot purchase/Sales of CHF affects both exchange position as well as Nostro A/C (Cash Position). However, forward
purchase/sale affects only the exchange position.

P a g e | 2.36 CA. Nagendra www.fmguru.org


FOREX

Transaction through demand draft (DD)


At the time of issuing DD Indian Bank will decrease its exchange position i.e. Bank book (to understand)

Nostro A/C will decrease only after presenting DD by Mr. S to his banker
If before presenting DD it will cancel, then Indian Bank will increase its exchange position.

India Switzerland
Think all transaction
for Indian Bank and
Indian Bank not for Mr. S and Mr. B Swiss Bank

Nostro Account of
3 Indian bank
2
DD in CHF 6 5
 1 CHF
Import DD
Mr. Buyer Mr. Seller

4
Pay by way of DD

After step-3 Indian bank will reduce exchange position.


After step-6 Nostro Account of Indian bank will decrease.
If DD canceled due to some reason the step 6 & 5 will not applicable and Mr Buyer will get money back and
Indian bank will increase Exchange position. No effect on Nostro A/C

Transaction through Bills Receivable (B/R):


. Bank buys bills receivable

India Switzerland

Present B/R at Maturity


Indian Bank 5 Swiss Bank

Nostro Account of
3 4 Indian bank

B/R in CHF  Export 6


1 CHF
Invoice in CHF
Mr. Seller Mr. Buyer

2
Received by way of B/R

After Step-3 Indian bank will increase exchange position.


After Step-5 & 6, Nostro A/C of Indian bank will increase.
If B/R dishonor, Indian bank will decrease exchange position and in this case there will be no effect on Nostro
Account.

www.fmguru.org CA. Nagendra P a g e | 2.37


Strategic Financial Management

Question No. - 12A [Nov-2005-old-7 Marks] [RTP-Nov-2008-old]


You as a dealer in foreign exchange have the following position in Swiss Francs on 31st October, 2004:
Swiss Francs
Balance in the Nostro A/c Credit 1,00,000
Opening Position Overbought 50,000
Purchased a bill on Zurich 80,000
Sold forward TT 60,000
Forward purchase contract cancelled 30,000
Remitted by TT 75,000
Draft on Zurich cancelled 30,000

What steps would you take, if you are required to maintain a credit Balance of Swiss Francs 30,000 in the Nostro
A/c and keep as overbought position on Swiss Francs 10,000?
Ans: Buy Spot TT for CHF 5000 and also Buy Forward TT for CHF 10,000

Question No. -12B [CWA-Dec-2004-1Marks]


An Indian bank wants to fund their Nostro A/C with US Correspondence bank by $5,00,000 against INR when Inter
bank rate is $1=47.20/50. The deal is struck and overseas bank’s Vostro A/C That is being maintained with the
Indian Bank will be credited by … … … … … .

Question No. -12C [CWA-June-2004-1 Marks]


A New York bank wants to fund their account called Vostro A/C, with an Indian bank by 10 million. What dollar
amount the New York bank would deposit in the Indian bank’s account called Nostro A/C maintained in New York
when Inter Bank rate is 1$ = 44.50-70

Cancellation of forward Contract:


Suppose a person enters forward contract on date “A” (i.e. today) for delivery on 3 month time. Due to some reason
contract cannot be performed and hence he approaches for cancellation. We are discussing the position for 4 different
situations.
3 month
2 month

Today before due date Due date after due date (Due date + 15 days)

Originally entered F. Contract for due date

1 3 4

1. Cancellation at 2 month time (i.e. 2


before due date)
2. Cancellation at due date
3. Cancellation after due date Applicable Rate for Cancellation
4. Automatic cancellation

P a g e | 2.38 CA. Nagendra www.fmguru.org


FOREX

Situation-1: Customer approaches to cancel forward contract before due date:


Bank enters sale contract, if original contract was for purchase (i.e. opposite contract), OR
Bank enters Purchase contract if original contract was for Sale (i.e. opposite contract), for due date exchange rate i.e. 1
month forward rate on date ‘B’.

Bank’s gain Customer’s gain

Gain to bank = Loss to customer or Loss to bank = Gain to customer


Gain/Loss to bank = Sale value under original contract – Purchase value under opposite contract
OR,
= sale value under opposite contract – Purchase value under original contract

Situation-2: Customer approaches to cancel forward contract on due date:


To cancel forward contract on date ‘C’, bank enters opposite contract at spot rate.

Bank’s gain Customer’s gain

Gain/Loss to both parties = same as above situation - ‘ ’

Situation-3: Customer approaches to cancel forward contract after due date:


Bank enters opposite contract at spot rate of cancellation date.

Bank’s gain Customer’s gain

Gain/Loss to both parties = same as above situation - ‘ ’


Situation-4: Customer doesn’t approaches to cancel forward contract. Hence automatic cancellation on due
date + 15 days:
Bank enters opposite contract at spot rate of cancellation date.
Bank’s gain Customer’s gain
Gain/Loss to both parties = same as above situation - ‘ ’

Question No. - 13A [Nov-2004-old-4marks]


A customer with whom the Bank had entered into 3 months forward purchase contract for Swiss Francs 1,00,000
at the rate of  36.25 comes to the bank after two months and requests cancellation of the contract. On this date,
the rates are:
Spot CHF 1 =  36.30 36.35
One month forward 36.45 36.52
Determine the amount of Profit or Loss to the customer due to cancellation of the contract.
Ans: Loss to the Customer =  27,000
Question No. - 13B [Nov-2002-old-6 Marks] [Same as Q-11A]
A customer with whom the Bank had entered into 3 months’ forward purchase contract for Swiss Francs ,
at the rate of  27.25 comes to the bank after 2 months and requests cancellation of the contract. On this date,
the rates, prevailing, are:
Spot SF 1 =  27.30 / 27.35
One month forward SF 1 =  27.45 / 27.52
What is the loss/gain to the customer on cancellation?
Ans: Loss to the Customer =  2,700

www.fmguru.org CA. Nagendra P a g e | 2.39


Strategic Financial Management

Question No. - 13C


ABC Ltd. booked a forward sale contract for US D 500,000 @ 46.20 due on 5 March. The customer did not contact
the bank on due date. However, on 10 March, the customer requests the Bank to cancel the contract. On this date,
spot rate is  46.28/46.35. What amount of gain/loss will be payable to/ receivable from customer?
Ans: Receivable from Customer =  75,000

Question No. - 13D


ABC Ltd. booked a forward sale contact for US D 200,000 due on 5 March, @  46.10. the customer did not
contact the bank on/ before/ after the date of maturity. Given the following spot rates, what amount of gain /
loss will be payable to/ receivable from customer?
19 March 46.17/46.25
20 March 46.18/46.26
21 March 46.19/46.27
Ans: Receivable from Customer =  32,000

Modification in forward contract:


Situation-I:
Some time customer enters forward contract to buy/sale the foreign currency but due to some reason (like defective goods)

later on customer need less amount than contracted amount.


Suppose customer originally entered forward purchase contract for $ 96,000 with bank but later on they need only $
50,000 and not $96,000. What he will do.
Solution:
Alternative-1
- Cancel the original forward contract on due date by entering the opposite contract.
- Purchase $ 50,000 at spot rate prevailing on that date (i.e. due date)

Alternative-2
- Purchase $96000 at committed rate and sell $46,000 at spot rate prevailing on due date

Situation-II:
Some time customer enters forward contract to buy/sale the foreign currency but due to some reason (like defective goods)

later on customer want to extend contract for some further period. What he will do.
Solution:
- Cancel earlier forward contract by entering opposite contract.
- Make fresh forward contract for the desired period.

Question No. -14A [RTP-Nov-2009-old] [RTP-May-2010-New]


On 30th June 2009 when a forward contract matured for execution you are asked by an importer customer to extend
the validity of the forward sale contract for US$ 10,000 for a further period of three months.
Contracted Rate US$1 = 41.87
The US Dollar quoted on 30.6.2009
Spot 40.4800/40.4900
Premium July 0.1100/0.1300
Premium August 0.2300/0.2500
Premium September 0.3500/0.3750
Calculate the cost for your customer in respect of the extension of the forward contract.
Rupee values to be rounded off to the nearest Rupee.
Margin 0.080% for Buying Rate
Margin 0.25% for Selling Rate
Ans: Cost to the customer =  14,200; Forward rate to be quoted to the customer is US $ 1 =  40.97

P a g e | 2.40 CA. Nagendra www.fmguru.org


FOREX

Question No. - 14B [June-2009-New-6 Marks] [Very Good Q.]


Your forex dealer had entered into a cross currency deal and had sold US $ 10,00,000 against EURO at US $ 1 = EUR
1.4400 for spot delivery.
However, later during the day, the market became volatile and the dealer in compliance with his management’s
guidelines had to square – up the position when the quotations were:
Spot US $ 1 INR 31.4300/4500
1 month margin 25/20
2 months margin 45/35
Spot US $ 1 EURO 1.4400/4450
1 month forward 1.4425/4490
2 months forward 1.4460/4530
What will be the gain or loss in the transaction?
Ans: Rupee Loss = 109,201.5

Question No. - 14C


X Ltd. a UK company, arranges on 1 January with a US supplier for purchase of goods costing US $ 96,000. X Ltd.
is supposed to pay 1 st July. X Ltd. arranged a forward exchange contract with its bank to Purchase $ 96, 000 six
months hence.
Later on the goods were found defective. As per agreement reached bet ween X Ltd. and US company, X Ltd. was
supposed to pay the US company only $ 50,000. X limited purchased only $ 50,000 though the contract was for $
96,000. Calculate the amount in sterling pound that X Ltd has to pay to its bank on the basis of following
information:
1 January Rate per sterling pound
Spot $ 1.5145 - 1.5155
6 month forward 0.95 - 0.85 cent premium
1 July spot $ 1.5100 - 1.5110
Ans: £ 33,343.97

Question No. -14D [CWA-June-2010-10 Marks]


VEDIKA TRADERS LTD. exports edible oils to Middle-East and African countries. In June the company exported an
assignment worth $ 5 million to Jambia. The payment for the same is expected to realize during the month of
September. For the company has entered into an option forward contract for delivery of $5 million over the month of
September.
The market quotes on June 30 at the time of entering into the contract were as follows:
June30. spot /$ 47.05/08
Forward 1 month 23/25 paise
2 month 47/49 paise
3 month 70/72 paise
On September’ , the company approached the bank for extension of the contract by another two months, that is for
delivery during the month of November.
The market quotes on September’ were as follows
spot /$ 47.58/60
Forward 1 month 18/20 paise
2 month 37/39 paise
3 month 55/57 paise
On November’ , the company approached the bank to cancel the forward contract. The exchange rates as on
November’ , were as follows:
spot /$ 47.97/99
Forward 1 month 16/18 paise
2 month 33/35 paise

www.fmguru.org CA. Nagendra P a g e | 2.41


Strategic Financial Management

You are required to calculate:


(i) The forward rate to be quoted to Laxmi Traders on June 30
(ii) The exchange rate to be quoted by the bank on September’ for the extension of the contract.
(iii) The amount of cash flows due to extension of the contract.
(iv) The exchange rate at which the forward contract to be cancelled on November’ .
(v) The amount of cash flows due to cancellation of the contract. (Ignore DEDAI margins for merchant
quotes.)
Ans: (i) 47.52; (ii) 47.95; (iii) 14 lakh; (iv) 48.17; (v) 11 lakh.

Question No. -14E [CWA-Dec-2006-8 Mks]


NBA Bank Ltd. Transacted on August 19, 2006 the following:
(i) Sold $ 10,00,000 two months forward to Alpha Manufacturing Co. Ltd. at 44.50
(ii) Purchase € , , two months forward from Beta Trading Co. Ltd. at 47.20;
On October 19, 2006, both the customers approached the bank. Alpha Manufacturing Co. wants the forward contract
to be cancelled while beta Trading co wants the contract to be extended by one month. The following exchange rates
prevailed on that day:
/$ /€
Spot 44.60/65 47.75/85
One month Forward 44.75/85 48.00/20
Based on the above information (ignore interest etc.) you are required to:
(i) Calculate the amount to be paid to or recovered from Alpha Manufacturing Co. due to the cancellation of
the forward contract.
(ii) Calculate the amount to be paid to or recovered from Beta Trading Company due to extension of the
forward contract.
Ans: (i) Amount to be paid to the customer =  1,00,000; (ii) Net amount to be collected from the customers = 5,00,000

LC (Letter of Credit) Arrangement


Question No. - 15A [Nov-2008-old-6 Marks] [Nov-1996-old-12 Marks]
Sun Ltd. in planning to import an equipment from Japan at a cost of 3,400 lakh yen. The company may avail loans at 18
per cent per annum with quarterly rests with which it can import the equipment. The company has also an offer from
Osaka branch of an India based bank extending credit of 180 days at 2 per cent per annum against opening of an
irrecoverable letter of credit.
Additional information:
Present exchange rate 100 = 340 yen
8 day’s forward rate 100 = 345 yen
Commission charges for letter of credit at 2 per cent per 12 months.
Advise the company whether the offer from the foreign branch should be accepted?
Ans: Option 1: Pmt = 1092.02 Lakh; Option 2: Pmt = 1006.28 Lakh.

Question No. – 15B [Study Mat.] [Same as Q- 15A ]


Beta Ltd. is planning to import a multi-purpose machine from Japan at a cost of 7,200 lakh yen. The company may
avail loans at 15%interest per annum with quarterly rests with which it can import the machine. The company has
also an offer from Tokyo branch of an India based bank extending credit of 180 days at 2% per annum against opening
of an irrecoverable letter of credit.
Additional information:
Present exchange rate 100 = 360 yen
8 day’s forward rate 100 = 365 yen
Commission charges for letter of credit at 2 per cent per 12 months.
Advise the company whether the offer from the foreign branch should be accepted?
Ans: Loan option: Pmt = 2152.81; LC: pmt = 2013.58

P a g e | 2.42 CA. Nagendra www.fmguru.org


FOREX

Question No. -15C [RTP-June-2009-old] [Same as Q-15A & 15B]


A Ltd. is planning to import a multi-purpose machine from France at a cost of 100 lakhs Francs. It can avail loans at
18% interest per annum with quarterly charge. However, there is an offer from Paris branch of an India based bank
extending credit of 180 days at 2% per quarter against opening of an irrevocable letter of credit.
Other Information:
Present exchange rate Re. 1.00 = 0.017 francs
8 days’ forward rate Re. . = . 6 francs
Commission charges for letter of credit at 2% per 12 months.
Advise whether the offer from the foreign branch should be accepted?
Ans: Loan option = 6423.67; Letter of credit = 6178; Forward contract = 6250.

Parallel Loan
When multinational corporations operating in various countries could not transfer funds among their subsidiaries
due to restriction in capital flows exercised by various governments, they came out with innovations of back-to-back
or parallel loans among themselves.
Parallel loan involve four parties, two multinational corporations and two subsidiaries in two different countries.

India USA

Indian Company US Subsidiary of


Indian company willing to fund Indian Company
its US Subsidiary $1,00,000

Interest in  Loan in  Loan in $


Interest in $

Indian Subsidiary of
US Company US Company
US company willing to fund its
Indian subsidiary in  currency
which is equivalent to $ 1,00,000

Figure showing Parallel loan between two companies

Question No. - 16A [Study Mat.]


MCDonnoughs Hamburger Company wishes to lend $ 5, 00,000 to its Japanese subsidiary. At the same time a
Yasufuku Heavy Industries is interested in making a medium-term loan of approximately the same amount to its
US subsidiary. The two parties are brought together by an Investment Bank for making parallel loans.
MCDonnoughs will lend $ 5, 00,000 to US subsidiary of Yasufuku for 4 Years @ 13 per cent (compounded
annually). Principal and interest are payable only at the end of next 4 years in dollars.
Yasufuku will lend to the Japanese subsidiary of McDonnoughs 70 million yen for 4 years at 10% (compounded
annually). The current exchange rate is 140 Yen per dollar. However the dollar is expected to decline by 5 yen
per dollar per year for each of next 4 years.
(a) What will be the dollar equivalent of Principal and interest payments to Yasufuku at the end of 4 years?
(b) What total Dollar McDonnoughs will receive after 4 years from the payment of principal and interest on
its loan by the US subsidiary of Yasafuku?
(c) Which party is better off with this deal? What would happen if the yen did not change in value?
Ans: (a) $ 854,058 (b) $815,237 (c) Yasufuku will be better off.

www.fmguru.org CA. Nagendra P a g e | 2.43


Strategic Financial Management

Cash management between Holding and Subsidiary


Question No. - 17A [RTP-Nov-2009-old] [Same as Q-19B]
Suppose you are a treasurer of XYZ plc in the UK. XYZ have two overseas subsidiaries, one based in Amsterdam and
one in Switzerland. The Dutch subsidiary has surplus Euros in the amount of 7,25,000 which it does not need for the
next three months but which will be needed at the end of that period (91 days). The Swiss subsidiary has a surplus of
Swiss Francs in the amount of 9,98,077 that, again, it will need on day 91. The XYZ plc in UK has a net balance of
£75,000 that is not needed for the foreseeable future.
Given the rates below, what is the advantage of swapping Euros and Swiss Francs into Sterling?
Spot Rate €/£ 0.6858 - 0.6869
91 day Pts 0.0037 0.0040
Spot Rate £/CHF 2.3295 - 2.3326
91 day Pts 0.0242 0.0228

Interest rates for the Deposits


91 day Interest Rate % pa
Amount of Currency
£ € CHF
0 – 100,000 1 ¼
100,001 – 500,000 2 1½ ¼
500,001 – 1,000,000 4 2 ½
Over 1,000,000 5.375 3 1
Ans: Total GBP on Individual Basis = £ 10,10,211, Total GBP on Swap to sterling = £ 10,13,488.61
[Hence advantage of swapping Euros and Swiss Francs into sterling = £ 3,277.61] [Assuming 365 days in a year]

Question No. -17B [May-2007-old-8 Marks] [CWA-June-2008-9 Marks] [RTP-May-2011] [Same as Q-19A]
AMK Ltd. an Indian based company has subsidiaries in U.S. and U.K.
Forecasts of surplus funds for the next 30 days from two subsidiaries are as below:
U.S. $12.5 million
U.K. £ 6 million

Following exchange rate information are obtained:

$/ £/

Spot 0.0215 0.0149

30 days forward 0.0217 0.0150


Annual borrowing/deposit rates (Simple) are available.
 6.4%/6.2%
$ 1.6%/1.5%
£ 3.9%/3.7%
The Indian operation is forecasting a cash deficit of 500 million.
It is assumed that interest rates are based on a year of 360 days.
(i) Calculate the cash balance at the end of 30 days period in  for each company under each of the following
scenarios ignoring transaction costs and taxes:

P a g e | 2.44 CA. Nagendra www.fmguru.org


FOREX

(a) Each company invests/finances its own cash balances/deficits in local currency independently.
(b) Cash balances are pooled immediately in India and the net balances are invested/borrowed for the 30
days period.
(ii) Which method do you think is preferable from the parent company’s point of view?

Ans: (i) (a) cash balance =  475,323; (b)  484,080; (ii) Immediate cash pooling is preferable.

Question No. - 17C [CWA-Dec-2002-8 Mks]


An American multinational corporation has subsidies whose cash positions for the month of September, 2002
are given below:
Swiss subsidiary : Cash surplus of SF 1, 50,00,000
Canadian subsidiary : Cash deficit of Can $ 2, 50,00,000
Cash deficit of 30,00,000 (UK
UK subsidiary : pound)

What are the cash requirements, if:


(i) Decentralized cash management is adopted?
(ii) Centralized cash management is adopted?
(Exchange rate: SF 1.48 / $, Can $ 1.58 / $, $ 1.5/ £)
Ans: (i) Decentralized management = $ 203,22,785;
(ii)Centralized management = $101,87,649

Netting of foreign exchange


Question No. -18A [RTP-Nov-2010-New]
Trueview Plc, a group of companies controlled from the United Kingdom includes subsidiaries in India, Malaysia
and the United States. As per the CFO’s forecast that, at the end of the june 2 010 the position of inter-company
indebtedness will be as follows:
 The Indian subsidiary will be owed 1,44,38,100 by the Malaysian subsidiary and will to owe the US
Subsidiary US$ 1,06,007.
 The Malaysian subsidiary will be owed MYR 14,43,800 by the US subsidiary and will owe it US$ 80,000.
Suppose you are head of central treasury department of the group and you are required to net off inter-company
balances as far as possible and to issue instructions for settlement of the net balances.
For this purpose, the relevant exchange rates may be assumed in terms of £ 1 are US$ 1.415; MYR 10.215;
68.10.
What are the net payments to be made in respect of the above balances?
Ans: Central treasury dept. will instruct to Malaysian Subsidiary to pay
Indian Sub = £1,27,209 and the US Subsidiary to pay the Indian subsidiary £9,887

Question No. - 18B [Nov-2006-old-4 Marks]


Following are the details of cash inflows and outflows in foreign currency denominations of MNP Co. an Indian
export firm, which have no foreign subsidiaries:
Currency Inflow Outflow Spot rate Forward rate
US $ 4,00,00,000 2,00,00,000 48.01 48.82

French Franc (FRF) 2,00,00,000 80,00,000 7.45 8.12

U.K. £ 3,00,00,000 2,00,00,000 75.57 75.98


Japanese Yen 1,50,00,000 2,50,00,000 3.20 2.40

www.fmguru.org CA. Nagendra P a g e | 2.45


Strategic Financial Management

(i) Determine the net exposure of each foreign currency in terms of Rupees.
(ii) Are any of the exposure positions offsetting to some extent
Ans: (i)Net exposure: $ = 16.2 Mill; FFr = 8.04 mill; £ = 4.10mill; Yen = 8 mill.
(iii) The exposure of Japanese yen position is being offset by a better forward rate

Expected exchange rate (Refer Portfolio)


Expected Value is an average value calculated using future data.
Expected value = ∑ (Different value Different Probability)
Refer Portfolio Management for further discussion on expected Value:

Question No. - 19A [May-2003-Old-6 Marks] [CWA-June-2008-4Mks]


In March 2003, the multinational industry makes the following assessment of dollar rates per British Pound to avail as
on 1.9.2003.
$/Pound Probability
1.6 0.15
1.7 0.20
1.8 0.25
1.9 0.20
2.0 0.20
1.00
(i) What is the expected spot rate for 1.9.2003?
(ii) If as of march, 2003, the 6-month forward rate is $ 1.80, should the firm sell forward its pound receivable due in
September 2003?
Ans: (i) $ 1.81; (ii) Firm do not sell pound under forward contract.

Forex with Derivative (All questions covered in Derivative chapter)


Question No. - 20A [Nov-2006-old-10 Marks] [Refer Derivative]
(a) XYZ Ltd. is an export oriented business house based in Mumbai. The Company invoices in customers’ currency. Its receipt of US $ 1,00,000 is due
on September 1, 2005.Market information as at June 1, 2005.
Exchange Rates Currency Futures
US $ /  US $ /  Contract size 4,72,000
Spot 0.02140 June 0.02126
1 Month Forward 0.02136 September 0.02118
3 Months Forward 0.02127

Initial Margin Interest Rates in India


June 10,000 7.50%
September 15,000 8.00%
On September 1, 2005 the spot rate US $Re. is 0.02133 and currency future rate is 0.02134. Comment, which of the following methods would be
most advantageous for XYZ Ltd?
(a) Using forward contract
(b) Using currency futures
(c) Not hedging currency risks.
It may be assumed that variation in margin would be settled on the maturity of the futures contract.
Ans: (a) Rcpt =  47,01,457; (b) Rcpt =  47,20,639; (c) Rcpt =  46,88,233.

Question No. – 20B [May-2010-Old-8 Marks] [RTP-May-2010-Old] [Refer Derivative]


Best of Luck Ltd, London will have to make a payment of $ 3,64,897 in six month’s time. It is currently st October. The company is considering the
various choices it has in order to hedge its transaction exposure.
Exchange rates
Spot rate $ 1.5617 - 1.5773
Six-months forward rate $ 1.5455 - 1.5609

P a g e | 2.46 CA. Nagendra www.fmguru.org


FOREX

Money Market rates: Borrow (%) Deposit (%)


US 6 4.5
UK 7 5.5

Foreign currency option prices (1 unit is £ 12,500):


Exercise Price Call option (March) Put option (March)
$1.70 / £ $0.037 $ 0.096
By making the appropriate calculations and ignoring time value of money (in case of premia) decide which of the following hedging alternatives is
the most attractive to best of luck ltd.
(a) Forward market
(b) Cash (money market)
(c) Currency options.

Question No.- 20C [May-2007-Old-16 Marks] [Refer Derivative]


XYZ Ltd. a US firm will need £ 3,00,000 in 180 days. In this connection, the following information is available:
Spot rate 1 £ = $ 2.00
180 days forward rate of £ as of today = $1.96
Interest rates are as follows:
U.K. US
180 days deposit rate 4.5% 5%
180 days borrowing rate 5% 5.5%
A call option on £ that expires in 180 days has an exercise price of $ 1.97 and a premium of $ 0.04.
XYZ Ltd. has forecasted the spot rates 180 days hence as below:
Future rate Probability
$ 1.91 25%
$ 1.95 60%
$ 2.05 15%
Which of the following strategies would be most preferable to XYZ Ltd.?
(a) a forward contract
(b) a money market hedge
(c) an option contract
(d) no hedging
Show calculations in each case.
Ans: (a) $5,88,000 (b) $ 6,05,741 (c) $5,94,900 (d) no hedging: 5,86500

Question No. -20D [CWA-Dec-2005-12 marks] [Refer Derivative]


ZENITH LTD (ZL) places an order to buy machinery with an American company. As per the agreement zenith Ltd will be paying $2,00,000 after 180
days. The company (ZL) considers to use:
(i) a forward hedge
(ii) a money market hedge,
(iii) an option hedge or
(iv) no hedge
The consultant of Zenith Ltd. collects and develops the following data/information as desired by the company, which can be used to assess the
alternative approaches for hedging:
(i) Spot rate of dollar as of today is 47/$
(ii) 180 day forward rate of dollar as of today is 47.50/$
(iii) Interest rats are as follows:
India US
180 days deposit rate (per annum) 7.5% 3%
180 day borrowing rate (per annum) 8.0% 4%
(Assume 360 days in a year)

(iv) Future spot rate in 180 days as estimated by the consultant is 47.75$.
(v) A call option on the dollar, which expires in 180 days has an exercise price of 47/$ and premium 0.52/$
(vi) A put option on the dollar, which expires in 180 days has an exercise price of 47.50 and premium 0.40/$.
Required:
Carry out a comparative analysis of various outcomes (rupee cost of import)/Alternatives and decide which of the alternatives is the most
attractive to zenith Ltd.
Ans: Price to be paid under various options: (i) 95 lakhs (ii) 96,31,511 (iii) 95,04,000;

www.fmguru.org CA. Nagendra P a g e | 2.47


Strategic Financial Management

(iv) 95,50,000; Forward hedge (Alternative-i) is best as it is cheapest.

Question No. -20E [CWA-Dec-2009-10 Marks] [Refer Derivative]


Wilson Ltd an Indian company has a payable of US$ 1,00,000 due in 3 months. The company is considering to cover the payable through the
following alternatives:
(i) Forward contract;
(ii) Money market; and
(iii) Option.
The following information is available with the company:
Exchange rate:
Spot /$ 45.50/45.55
3M Forward 40/45

Interest rates (%): Per Annum


US 4.5/5.0 (Deposit/Borrow)
India 10.0/11.0 (Deposit/Borrow)
Call option on $ with a strike price of 46.00 is available at a premium of 0.10/$. Put option on $ with a strike price of 46.00 is available with a
premium of 0.05/$.
Treasury department of the company forecasted the future spot rate after 3 months to be:
Spot rate after 3-m Probability
45.60/$ 0.10
46.00/$ 0.60
 46.40/$ 0.30
You are required to suggest the best alternative of hedging.
Ans: (i) Forward hedge: 46,00,000; (ii) MMO: 46,28,195; (iii) Option hedge:

P a g e | 2.48 CA. Nagendra www.fmguru.org


FOREX

www.fmguru.org CA. Nagendra P a g e | 2.49


Strategic Financial Management

P a g e | 2.50 CA. Nagendra www.fmguru.org

Potrebbero piacerti anche